Quantity A

  • May 2020
  • PDF

This document was uploaded by user and they confirmed that they have the permission to share it. If you are author or own the copyright of this book, please report to us by using this DMCA report form. Report DMCA


Overview

Download & View Quantity A as PDF for free.

More details

  • Words: 25,661
  • Pages: 119
Quantity A: (-6)4 Quantity B: (-6)5 A. B. C. D.

if the quantity A is greater; if the quantity B is greater; if the two quantities are equal; if the relationship cannot be determined from the information given.

Ans : A Quantity A: Time to travel 95 miles at 50 miles per hour Quantity B: Time to travel 125 miles at 60 miles per hour A. B. C. D.

Quantity A is greater Quantity A equals Quantity B Quantity B is greater Relationship Indeterminate

Ans : C Quantity A: (9/13)2 Quantity B: (9/13)1/2 A. B. C. D.

Quantity A equals Quantity B Relationship Indeterminate Quantity B is greater Quantity A is greater

Ans : C Quantity A: 4 / 100 Quantity B: 0.012 / 3 A. B. C. D.

Quantity B is greater Quantity A equals Quantity B Quantity A is greater Relationship Indeterminate

Ans : C x = 2y + 3 y = -2 Quantity A: x Quantity B: -1 A. B. C. D.

if the quantity in Column A is greater if the quantity in Column B is greater if the two quantities are equal if the relationship cannot be determined from the information given

Ans : C x + 2y > 8 Quantity A: 2x + 4y Quantity B: 20 A. B. C. D.

if the quantity in Column A is greater if the quantity in Column B is greater if the two quantities are equal if the relationship cannot be determined from the information given.

Ans : D Quantity A: The number of months in 7 years Quantity B: The number of days in 12 weeks A. B. C. D.

if the quantity in Column A is greater if the quantity in Column B is greater if the two quantities are equal if the relationship cannot be determined from the information given

Ans : C Quantity A: 1-1/27 Quantity B: 8/9 + 1/81 A. B. C. D.

if the quantity in is greater if the quantity in is greater if the two quantities are equal if the relationship cannot be determined from the information given.

Ans : A r/>s/>0/> Quantity A: rs/r Quantity B: rs/s A. B. C. D.

if the quantity A is greater if the quantity B is greater if the two quantities are equal if the relationship cannot be determined from the information given.

Ans : B Quantity A: 0.83 Quantity B: 0.81/3 A. Quantity B is greater

B. Relationship Indeterminate C. Quantity A is greater D. Quantity A equals Quantity B Ans : A 11. Ans : D 12. Quantity A: (0.82)2(0.82)3 Quantity B:(0.82)6 A. if the quantity in Column A is greater; B. if the quantity in Column B is greater; C. if the two quantities are equal; D. if the relationship cannot be determined from the information given. Ans : A 13. For all real numbers a, let a* = 1 - a. Quantity A: ((-1)*)* Quantity B: 2* A. B. C. D.

if the quantity in Column A is greater; if the quantity in Column B is greater; if the two quantities are equal; if the relationship cannot be determined from the information given.

Ans : C 14. Quantity A: (x - 1)(x)(x + 1) Quantity B:(x)(x)(x) A. if the quantity in Column A is greater; B. if the quantity in Column B is greater; C. if the two quantities are equal; D. if the relationship cannot be determined from the information given. Ans : D 15. Quantity A: (3 x 4 x 17) / (121 x 100) Quantity B: (4 x 5 x 19) / (1000 x 121) A. Quantity A is greater B. Quantity A equals Quantity B C. Relationship Indeterminate D. Quantity B is greater A 16. Consider a triangle PQR. Quantity A: length of PQ + length of QR Quantity B: length of PR

A. B. C. D.

Quantity A is greater Quantity B is greater Relationship Indeterminate Quantity A equals Quantity B

Ans : A 17. Quantity A: (27 - 13) (296 + 534) Quantity B: (27 + 13) (534 + 296) A. Quantity B is greater B. Quantity A equals Quantity B C. Relationship Indeterminate D. Quantity A is greater Ans : D 18. Quantity A: A = 1.1 Quantity B: 12.11/2 A. Relationship Indeterminate B. Quantity B is greater C. Quantity A equals Quantity B D. Quantity A is greater Ans : B 19. 100 < y < 200 and 100 < z < 210 Quantity A: y Quantity B: z A. B. C. D.

Quantity A is greater Quantity A equals Quantity B Quantity B is greater Relationship Indeterminate

Ans : D 20. y2 + z2 = 34 and yz = 15 Quantity A: y2 + 2yz + z2 Quantity B: (y + z)2 A. B. C. D.

Quantity B is greater Relationship Indeterminate Quantity A is greater Quantity A equals Quantity B

Ans : D

Quantitative Comparisons 21. Consider a rectangle. The length of its shorter side is 8, and the length of its diagonal is 16. 30o Quantity B: measure of angle formed by diagonal and shorter side Quantity A:

A. Relationship Indeterminate B. Quantity A equals Quantity B C. Quantity A is greater D. Quantity B is greater Ans : D 22. Quantity A: (y + 5)2 Quantity B: (y - 5)2 A. Quantity B is greater B. Relationship Indeterminate C. Quantity A equals Quantity B D. Quantity A is greater Ans : B 23. Quantity A: (1/25)1/2 + (1/144)1/2 Quantity B: [(1/25) + (1/144)]1/2 A. Relationship Indeterminate B. Quantity A is greater C. Quantity B is greater D. Quantity A equals Quantity B Ans : A 24. y2 + z2 = 34 and yz = 15 Quantity A: y2 + 2yz + z2 Quantity B: (y + z)2 A. B. C. D.

Quantity A is greater Relationship Indeterminate Quantity A equals Quantity B Quantity B is greater

Ans : C 25. 100 < y < 200 and 100 < z < 210 Quantity A: y Quantity B: z

A. B. C. D.

Quantity A is greater Quantity A equals Quantity B Quantity B is greater Relationship Indeterminate

Ans : D 26. Quantity A: (y + 5)2 Quantity B: (y - 5)2 A. Quantity A equals Quantity B B. Quantity A is greater C. Relationship Indeterminate D. Quantity B is greater Ans : C 27. Consider a rectangle. The length of its shorter side is 8, and the length of its diagonal is 16. Quantity A: 30o Quantity B: measure of angle formed by diagonal and shorter side A. B. C. D.

Quantity A is greater Quantity A equals Quantity B Quantity B is greater Relationship Indeterminate

Ans : C 28. The sum of three consecutive even numbers is 18. Quantity A: Their average Quantity B: 6 A. B. C. D.

Relationship Indeterminate Quantity A is greater Quantity A equals Quantity B Quantity B is greater

Ans : C 29. x - y > 10 Quantity A: y - x Quantity B: 12 A. B. C. D.

Quantity B is greater Quantity A is greater Quantity A equals Quantity B Relationship Indeterminate

Ans : A 30. x = 0, y > 0 Quantity A: xy Quantity B: yx A. B. C. D.

Quantity A equals Quantity B Quantity A is greater Quantity B is greater Relationship Indeterminate

Ans : C

Quantitative Comparisons

31. Diagram is illustrative and is not drawn to scale. Quantity A: Measure

of angle 3 - Measure of angle 2 Quantity B: Measure of angle 5 - Measure of angle 6 A. B. C. D.

Relationship Indeterminate Quantity A equals Quantity B Quantity B is greater Quantity A is greater

Ans : B 32. Quantity A: 29 Quantity B: 92 A. Quantity B is greater B. Quantity A is greater C. Relationship Indeterminate D. Quantity A equals Quantity B Ans : B

33. 0 < -x < 10 11 < -y < 20 Quantity A: x Quantity B: y A. B. C. D.

Relationship Indeterminate Quantity A equals Quantity B Quantity B is greater Quantity A is greater

Ans : D

34. Diagram is illustrative and is not drawn to scale. Given angles 1 and 2 are equal, length of side AB = x, length of side BC = y, length of side AC = z. Quantity A: x + y Quantity B: y + z A. B. C. D.

Quantity A is greater Quantity A equals Quantity B Quantity B is greater Relationship Indeterminate

Ans : B

35. Diagram is illustrative and is not drawn to scale. In triangle ABC, AB = AC and measure of angle 1 = 100o.

Quantity A: Measure of angle 2 + Measure of angle 3 Quantity B: 90o A. B. C. D.

Quantity B is greater Relationship Indeterminate Quantity A is greater Quantity A equals Quantity B

Ans : A 36. x and y are both positive and x/y > 5 Quantity A: 0.2x Quantity B: y A. B. C. D.

Quantity A is greater Quantity B is greater Relationship Indeterminate Quantity A equals Quantity B

Ans : A

37. Diagram is illustrative and is not drawn to scale. Given AB = AC and angle BAC = 60o Quantity A: Length of side AB Quantity B: Length of side BC A. B. C. D.

Quantity A equals Quantity B Quantity B is greater Relationship Indeterminate Quantity A is greater

Ans : A 38. y2 = 36 Quantity A: y Quantity B: 6

A. B. C. D.

Relationship Indeterminate Quantity A is greater Quantity B is greater Quantity A equals Quantity B

Ans : A

39. Diagram is illustrative and is not drawn to scale. Quantity A: Measure of angle 1 + Measure of angle 2 + Measure of angle 4 Quantity B: 180o A. B. C. D.

Relationship Indeterminate Quantity A is greater Quantity B is greater Quantity A equals Quantity B

Ans : D

40. Diagram is illustrative and is not drawn to scale. In triangle ABC, angle A = 60o and AB = AC. Quantity A: Measure of angle 1 + Measure of angle 2 Quantity B: 120o A. B. C. D.

Relationship Indeterminate Quantity A is greater Quantity A equals Quantity B Quantity B is greater

Ans : C

Quantitative Comparisons

41. Diagram is illustrative and is not drawn to scale. Quantity A: Measure

of angle 2 + Measure of angle 3

o

Quantity B: 180 A. B. C. D.

Quantity B is greater Quantity A is greater Quantity A equals Quantity B Relationship Indeterminate

Ans : C

42. Diagram is illustrative and is not drawn to scale. AB is the diameter of the circle. Quantity A: Measure of angle 1 Quantity B: Measure of angle 2 A. B. C. D.

Relationship Indeterminate Quantity A is greater Quantity B is greater Quantity A equals Quantity B

Ans : D

43. Diagram is illustrative and is not drawn to scale. Quantity A: Measure of angle 1 + Measure of angle 3 Quantity B: Measure of angle 2 + Measure of angle 4 A. B. C. D.

Relationship Indeterminate Quantity A is greater Quantity A equals Quantity B Quantity B is greater

Ans : A

44. Diagram is illustrative and is not drawn to scale. In triangle ABC, AB = AC and measure of angle 1 = 100o. Quantity A: Measure of angle 2 + Measure of angle 3 Quantity B: 90o A. B. C. D.

Quantity B is greater Quantity A equals Quantity B Relationship Indeterminate Quantity A is greater

Ans : A

45. Diagram is illustrative and is not drawn to scale. Given angles 1 and 2 are equal, length of side AB = x, length of side BC = y, length of side AC = z. Quantity A: x + y Quantity B: y + z A. B. C. D.

Quantity B is greater Quantity A equals Quantity B Quantity A is greater Relationship Indeterminate

Ans : B 46. x and y are both positive and x/y > 5 Quantity A: 0.2x Quantity B: y A. B. C. D.

Quantity B is greater Relationship Indeterminate Quantity A equals Quantity B Quantity A is greater

Ans : D 47. yz < 0 Quantity A: (y - z)2 Quantity B: y2 + z2 A. B. C. D.

Quantity A is greater Quantity B is greater Quantity A equals Quantity B Relationship Indeterminate

Ans : A 48. For any positive integer n, n! is the product of all positive integers less than or equal to n.

Quantity A: 20! / 17! Quantity B: 80! / 78! A. B. C. D.

Quantity A is greater Quantity B is greater Quantity A equals Quantity B Relationship Indeterminate

Ans : A 49. 2 < z < 4 Quantity A: π2z3 Quantity B: π3z2 A. B. C. D.

Quantity A is greater Quantity B is greater Quantity A equals Quantity B Relationship Indeterminate

Ans : D 50. Amy, Beth and Charlie divided a pizza amongst themselves. Amy took 30% of the pizza and ate (3/4) of what she took. Beth took 20% of the pizza. Charlie ate (2/5) of what he took. Quantity A: The amount Amy ate Quantity B: The amount Charlie ate A. B. C. D.

Quantity A is greater Quantity B is greater Quantity A equals Quantity B Relationship Indeterminate

Ans : A

ative Comparisons 51. p > 0 > q Quantity A:

p+q Quantity B: pq A. The quantity in Column A is greater. B. The quantity in Column B is greater. C. The quantities are equal.

D.

The relationship

cannot be determined from the information given.

Ans : D 52. Quantity A: The average (arithmetic mean) of x and y Quantity B: The average (arithmetic mean) of x - 1 and y + 1 A. The quantity in Column A is greater. B. The quantity in Column B is greater. C. The quantities are equal. D. The relationship cannot be determined from the information given. Ans : C 53. The integer (x - 1) is a prime number between 40 and 50. Quantity A: The sum of all different prime factors of x Quantity B: 14 A. B. C. D.

The quantity in Column A is greater. The quantity in Column B is greater. The quantities are equal. The relationship cannot be determined from the information given.

Ans : B 54. A < C B>D>0 Quantity A: A - B Quantity B: C - D A. B. C. D.

The quantity in Column A is greater. The quantity in Column B is greater. The quantities are equal. The relationship cannot be determined from the information given.

Ans : B 55. In a particular jellybean jar, the number of red jellybeans exceeds the number of white ones by a ratio of 3:2. If two red jellybeans were removed, the ratio of red to white jellybeans would be 1:1. Quantity A: The number of white jellybeans in the jar Quantity B: 4 A. The quantity in Column A is greater. B. The quantity in Column B is greater. C. The quantities are equal. be determined from the information given. Ans : C

D. tionship cannot

The relauantitative Section : Quantitative Ability Directions: Solve each CAT sample quantitative ability problem and indicate the best of the answer choices given.. Numbers: All numbers used are real numbers. Figures: A figure accompanying a CAT sample quantitatove ability problem solving question is intended to provide information useful in solving the problem. Figures are drawn as accurately as possible EXCEPT when it is stated in a specific problem that its figure is not drawn to scale. Straight lines may sometimes appear jagged. All figures lie on a plane unless otherwise indicated. Following are some CAT sample quantitative ability questions. 1. A rectangle is 14 cm long and 10 cm wide. If the length is reduced by x cms and its width is increased also by x cms so as to make it a square then its area changes by : A. 4 B. 144 C. 12 D. 2 E. None of the above. Ans : A 2. A motorcycle stunts man belonging to a fair, rides over the vertical walls of a circular well at an average speed of 54 kph for 5 minutes. If the radius of the well is 5 meters then the distance traveled is: A. 2.5 kms B. 3.5 kms C. 4.5 kms D. 5.5 kms E. None of the above Ans : C 3. If 1 cm on a map corresponds to an actual distance of 40 kms. And the distance on the map between Bombay and Calcutta is 37.5 cms., the actual distance between them is : A. 375 kms B. 3750 kms C. 1500 kms D. 1375 kms E. None of the above

Ans : C 4. A box contains 90 mts each of 100 gms and 100 bolts each of 150 gms. If the entire box weighs 35.5 kg., then the weight of the empty box is : A. 10 kg B. 10.5 kg C. 11 kg D. 11.5 kg E. None of the above Ans : D 5. If the radius of a circle is increased by 20% then the area is increased by : A. 44% B. 120% C. 144% D. 40% E. None of the above Ans : A 6. Tom, Dick and Harry went for lunch to a restaurant. Tom had $100 with him, Dick had $60 and Harry had $409. They got a bill for $104 and decided to give a tip of $16. They further decided to share the total expenses in the ratio of the amounts of money each carried. The amount of money which Tom paid more than what Harry paid is A. 120 B. 200 C. 60 D. 24 E. 36 Ans : E 7. A plot of land is in the shape of a trapezium whose dimensions are given in the figure below :

Hence the perimeter of the field is A. 50 m B. 64 m C. 72 m D. 84 m E. None of the above Ans : c

8. Four concentric ( having the same center ) circles with radii, x, 2x, 3x and 4x are drawn to form two rings A and B as shown in the figure.

Ratio of the area of inner ring A to the area of outer ring B is A. 1 : 2 B. 1 : 4 C. 2 : 3 D. 3 : 7 E. None of the above Ans : D 9. If 3/p = 6 and 3/q = 15 then p - q = ? A. 1/3 B. 2/5 C. 3/10 D. 5/6 E. None of the above Ans : C 10. A father is three times as old as his son. After fifteen years the father will be twice as old as his son's age at that time. Hence the father's present age is A. 36 B. 42 C. 45 D. 48 E. None of the above Ans : C

titative Section : Quantitative Ability 11. (1/4)3 + (3/4)3 + 3(1/4)(3/4)(1/4 + 3/4) =? A. 1/64 B. 27/64 C. 49/64 D. 0 E. 1 Ans : E

12. If the area of two circles are in the ratio 169 : 196 then the ratio of their radii is A. 10 : 11 B. 11 : 12 C. 12 : 13 D. 13 : 14 E. None of the above Ans : D 13. A semi-circle is surmounted on the side of a square. The ratio of the area of the semi-circle to the area of the square is

A. B. C. D. E.

1:2 2:p p:8 8:p None of the above

Ans : C 14. Which of the following is the greatest ? A. 40% of 30 B. 3/5 of 25 C. 6.5% of 200 D. Five more than the square of 3 E. 1/2-4 Ans : E 15. Two identical taps fill 2/5 of a tank in 20 minutes. When one of the taps goes dry in how many minutes will the remaining one tap fill the rest of the tank ? A. 5 minutes B. 10 minutes C. 15 minutes D. 20 minutes E. None of the above Ans : C 16. If the value of XYZ Company stock drops from $25 per share to $21 per share, what is the percent of the decrease? A. 4 B. 8

C. 12 D. 16 E. 20 Ans : D 17. If a building b feet high casts a shadow f feet long, then, at the same time of day, a tree t feet high will cast a shadow how many feet long? A. ft/b B. fb/t C. b/ft D. tb/f E. t/fb Ans : A 18. If x, y, and z are consecutive negative integers, and if x > y > z, which of the following must be a positive odd integer? A. xyz B. (x - y) (y - z) C. x - yz D. x(y + z) E. x + y + z Ans : B 19. At a certain ice cream parlor, customers can choose among five different ice cream flavors and can choose either a sugar cone or a waffle cone. Considering both ice cream flavor and cone type, how many distinct triple-scoop cones with three different ice cream flavors are available? A. 12 B. 16 C. 20 D. 24 E. 30 Ans : C 20. What is the greatest value of a positive integer n such that 3n is a factor of 1815? A. 15 B. 18 C. 30 D. 33 E. 45 Ans : C

Quantitative Section : Quantitative Ability

21. If .2t = 2.2 - .6s and .5s = .2t + 1.1, then s = A. 1 B. 3 C. 10 D. 11 E. 30 Ans : B 22. Five years ago, Beth's age was three times that of Amy. Ten years ago, Beth's age was one half that of Chelsea. If C repre- sents Chelsea's current age, which of the following represents Amy's current age? A. c/6 + 5 B. 2c C. (c-10)/3 D. 3c-5 E. 5c/3 - 10 Ans : A 23. A portion of $7200 is invested at a 4% annual return, while the remainder is invested at a 5% annual return. If the annual income from both portions is the same, what is the total income from the two investments? A. $160 B. $320 C. $400 D. $720 E. $1,600 Ans : B 24. An empty swimming pool can be filled to capacity through an inlet pipe in 3 hours, and it can be completely drained by a drainpipe in 6 hours. If both pipes are fully open at the same time, in how many hours will the empty pool be filled to capacity? A. 4 B. 4.5 C. 5 D. 5.5 E. 6 Ans : E 25. If r = (3p + q)/2 and s = p - q, for which of the following values of p would r2 = s2? A. 1q/5 B. 10 - 3q/2 C. q - 1 D. 3q E. 9q/2 - 9 Ans : A

26. At 10 a.m. two trains started traveling toward each other from stations 287 miles apart. They passed each other at 1:30 p.m. the same day. If the average speed of the faster train exceeded the average speed of the slower train by 6 miles per hour, which of the following represents the speed of the faster train, in miles per hour? A. 38 B. 40 C. 44 D. 48 E. 50 Ans : C 27. On the xy-coordinate plane, points A and B both lie on the circumference of a circle whose center is O, and the length of AB equals the circle's diameter. If the (x,y) coordinates of O are (2,1) and the (x,y) coordinates of B are (4,6), what are the (x,y) coordinates of A? A. (3, 3/2) B. (1, 2/2) C. (0, -4) D. (2/2, 1) E. (-1, -2/2) Ans : C 28. If a rectangle's length and width are both doubled, by what percent is the rectangle's area increased? A. 50 B. 100 C. 200 D. 300 E. 400 Ans : D 29. A rectangular tank 10" by 8" by 4" is filled with water. If all of the water is to be transferred to cube-shaped tanks, each one 3 inches on a side, how many of these smaller tanks are needed? A. 9 B. 12 C. 16 D. 21 E. 39 Ans : B 30. Point Q lies at the center of the square base (ABCD) of the pyramid pictured above. The pyramid's height (PQ) measures exactly one half the length of each edge of its base, and point E lies exactly halfway between C and D along one edge of the base. What is the ratio of the surface area of any of the pyramid's four triangular faces to the surface area of the shaded triangle?

A. B. C. D. E.

3 :√2 √5:1 4√3:3 2√2:1 8:√5

Ans : D

Quantitative Section : Quantitative Ability Average 31. The average wages of a worker during a fortnight comprising 15 consecutive working days was Rs.90 per day. During the first 7 days, his average wages was Rs.87/day and the average wages during the last 7 days was Rs.92 /day. What was his wage on the 8th day? A. 83 B. 92 C. 90 D. 97 Ans : D 32. The average of 5 quantities is 6. The average of 3 of them is 8. What is the average of the remaining two numbers? A. 6.5 B. 4 C. 3 D. 3.5 Ans : C 33. The average temperature on Wednesday, Thursday and Friday was 250. The average temperature on Thursday, Friday and Saturday was 240. If the temperature on Saturday was 270, what was the temperature on Wednesday? A. 240 B. 210 C. 270 D. 300 Ans : D 34. The average age of a group of 12 students is 20years. If 4 more students join the group, the average age increases by 1 year. The average age of the new students is A. 24 B. 26 C. 23 D. 22

Ans : A 35. When a student weighing 45 kgs left a class, the average weight of the remaining 59 students increased by 200g. What is the average weight of the remaining 59 students? A. 57 kgs B. 56.8 kgs C. 58.2 kgs D. 52.2 kgs Ans : A 36. Three math classes: X, Y, and Z, take an algebra test. The average score in class X is 83. The average score in class Y is 76. The average score in class Z is 85. The average score of all students in classes X and Y together is 79. The average score of all students in classes Y and Z together is 81. What is the average for all the three classes? A. B. C. D.

81 81.5 82 84.5

Ans : B 37. The average weight of a class of 24 students is 36 years. When the weight of the teacher is also included, the average weight increases by 1kg. What is the weight of the teacher? A. 60 kgs B. 61 kgs C. 37 kgs D. None of these Ans : B 38. The average of 5 quantities is 10 and the average of 3 of them is 9. What is the average of the remaining 2? A. 11 B. 12 C. 11.5 D. 12.5 Ans : C 39. The average age of a family of 5 members is 20 years. If the age of the youngest member be 10 years then what was the average age of the family at the time of the birth of the youngest member?

A. B. C. D.

13.5 14 15 12.5

Ans : D 40. A student finds the average of 10 positive integers. Each integer contains two digits. By mistake, the boy interchanges the digits of one number say ba for ab. Due to this, the average becomes 1.8 less than the previous one. What was the difference of the two digits a and b? A. 8 B. 6 C. 2 D. 4 Ans : C 41. Average cost of 5 apples and 4 mangoes is Rs. 36. The average cost of 7 apples and 8 mangoes is Rs. 48. Find the total cost of 24 apples and 24 mangoes. A. 1044 B. 2088 C. 720 D. 324 Ans : B

Quantitative Section : Quantitative Ability Interest 41. A father left a will of Rs.35 lakhs between his two daughters aged 8.5 and 16 such that they may get equal amounts when each of them reach the age of 21 years. The original amount of Rs.35 lakhs has been instructed to be invested at 10% p.a. simple interest. How much did the elder daughter get at the time of the will? A. Rs. 17.5 lakhs B. Rs. 21 lakhs C. Rs. 15 lakhs D. Rs. 20 lakhs Ans : B 42. What will Rs.1500 amount to in three years if it is invested in 20% p.a. compound interest, interest being compounded annually? A. 2400 B. 2592 C. 2678 D. 2540

Ans : B 43. If a sum of money grows to 144/121 times when invested for two years in a scheme where interest is compounded annually, how long will the same sum of money take to treble if invested at the same rate of interest in a scheme where interest is computed using simple interest method? A. 9 years B. 22 years C. 18 years D. 33 years Ans : B 44. The population of a town was 3600 three years back. It is 4800 right now. What will be the population three years down the line, if the rate of growth of population has been constant over the years and has been compounding annually? A. 6000 B. 6400 C. 7200 D. 9600 Ans : B 45. A man invests Rs.5000 for 3 years at 5% p.a. compound interest reckoned yearly. Income tax at the rate of 20% on the interest earned is deducted at the end of each year. Find the amount at the end of the third year. A. 5624.32 B. 5630.50 C. 5788.125 D. 5627.20 Ans : A 46. The difference between the compound interest and the simple interest on a certain sum at 12% p.a. for two years is Rs.90. What will be the value of the amount at the end of 3 years? A. 9000 B. 6250 C. 8530.80 D. 8780.80 Ans : D 47. Vijay invested Rs.50,000 partly at 10% and partly at 15%. His total income after a year was Rs.7000. How much did he invest at the rate of 10%? A. Rs.40,000 B. Rs.40,000 C. Rs.12,000 D. Rs.20,000

Ans : B 48. A sum of money invested for a certain number of years at 8% p.a. simple interest grows to Rs.180. The same sum of money invested for the same number of years at 4% p.a. simple interest grows to Rs.120 only. For how many years was the sum invested? A. 25 years B. 40 years C. 33 years and 4 months D. Cannot be determined Ans : A 49. How long will it take for a sum of money to grow from Rs.1250 to Rs.10,000, if it is invested at 12.5% p.a simple interest? A. 8 years B. 64 years C. 72 years D. 56 years Ans : D 50. Rs. 5887 is divided between Shyam and Ram, such that Shyam's share at the end of 9 years is equal to Ram's share at the end of 11 years, compounded annually at the rate of 5%. Find the share of Shyam. A. 2088 B. 2000 C. 3087 D. None of these Ans : C

Quantitative Section : Quantitative Ability Geometry 51. Find the coordinates of the point which divides the line joining (5, -2) and (9, 6) internally in the ratio 1 : 3. A. (6, 0) B. (6, 3) C. (0, 6) D. (3, 6) Ans : A 52. Find the number of triangles in an octagon. A. 326 B. 120 C. 56

D. Cannot be determined Ans : C 53. Find the equation of a line whose intercepts are twice of the line 3x - 2y - 12 = 0 A. 3x - 2y = 24 B. 2x - 3y = 12 C. 2x - 3y = 24 D. None of these Ans : A 54. Find the area of the sector covered by the hour hand after it has moved through 3 hours and the length of the hour hand is 7cm. A. 77 sq.cm B. 38.5 sq.cm C. 35 sq.cm D. 70 sq.cm Ans : B 55. Find the area of the triangle whose vertices are (-6, -2), (-4, -6), (-2, 5). A. 36 B. 18 C. 15 D. 30 Ans : C 56. A stairway 10ft high is such that each step accounts for half a foot upward and onefoot forward. What distance will an ant travel if it starts from ground level to reach the top of the stairway? A. 30 ft B. 33 ft C. 10 ft D. 29 ft Ans : D 57. Each interior angle of a regular polygon is 120 degrees greater than each exterior angle. How many sides are there in the polygon? A. 6 B. 8 C. 12 D. 3 Ans : C 58. What is the area of the largest triangle that can be fitted into a rectangle of length 'l' units and width 'w' units?

A. B. C. D.

lw/3 (2lw)/3 (3lw)/4 (lw)/2

Ans : D 59. Which of the following is inCorrect? A. An incentre is a point where the angle bisectors meet. B. The median of any side of a triangle bisects the side at right angle. C. The point at which the three altitudes of a triangle meet is the orthocentre D. The point at which the three perpendicular bisectors meet is the centre of the circumcircle. Ans : B 60. A and B are two points with the co-ordinates (-2, 0) and (0, 5). What is the length of the diagonal AC if AB form one of the sides of the square ABCD? A. units B. units C. units D. units Ans : B

Quantitative Section : Quantitative Ability Geometry 61. What is the measure of the circum radius of a triangle whose sides are 9, 40 and 41? A. 6 B. 4 C. 24.5 D. 20.5 Ans : D 62. If the sum of the interior angles of a regular polygon measures up to 1440 degrees, how many sides does the polygon have? A. 10 sides B. 8 sides C. 12 sides D. 9 sides Ans : A

63. If ABC is a right angle triangle with angle A = 900 and 2s = a + b + c, where a > b > c where notations have their usual meanings, then which one of the following is Correct? A. (s - b) (s - c) > s (s - a) B. (s - a) (s - c) > s (s - b) C. (s - a) (s - b) < s (s - c) D. 4s (s - a) (s - b) (s - c) = bc Ans : C 64. What is the measure of in radius of the triangle whose sides are 24, 7 and 25? A. 12.5 B. 3 C. 6 D. None of these Ans : B 65. What is the circum radius of a triangle whose sides are 7, 24 and 25 respectively? A. 18 B. 12.5 C. 12 D. 14 Ans : B

Quantitative Ability : Menstruation 66. A regular hexagon is inscribed in a circle of radius r cms. What is the perimeter of the regular hexagon? A. 3r B. 6r C. r D. 9r Ans : B 67. A 4 cm cube is cut into 1 cm cubes. What is the percentage increase in the surface area after such cutting? A. 4% B. 300% C. 75% D. 400% Ans : B 68. If the diagonal and the area of a rectangle are 25 m and 168 m2, what is the length of the rectangle? A. 17 m

B. 31 m C. 12 m D. 24 m Ans : D 69. The surface area of the three coterminous faces of a cuboid are 6, 15, 10 sq.cm respectively. Find the volume of the cuboid. A. 30 B. 20 C. 40 D. 35 Ans : A 70. If each interior angle of a regular polygon is 150 degrees, then it is A. Octagon B. Decagon C. Dodecagon D. Tetrahedron Ans : C

Quantitative Section : Quantitative Ability Menstruation 71. A 5 cm cube is cut into as many 1 cm cubes as possible. What is the ratio of the surface area of the larger cube to that of the sum of the surface areas of the smaller cubes? A. 1 : 6 B. 1 : 5 C. 1 : 25 D. 1 : 125 Ans : B 72. If the sides of a triangle measure 72, 75 and 21, what is the measure of its in radius? A. 37.5 B. 24 C. 9 D. 15 Ans : C

73. The circumference of the front wheel of a cart is 30 ft long and that of the back wheel is 36 ft long. What is the distance travelled by the cart, when the front wheel has done five more revolutions than the rear wheel? A. 20 ft B. 25 ft C. 750 ft D. 900 ft Ans : D 74. The area of a square field is 24200 sq m. How long will a lady take to cross the field diagonally at the rate of 6.6 km/hr? A. 3 minutes B. 2 minutes C. 2.4 minutes D. 2 minutes 40 seconds Ans : B

Quantitative Ability : Trignometry 75. a and b are the lengths of the base and height of a right angled triangle whose hypotenuse is h. If the values of a and b are positive integers, which of the following cannot be a value of the square of the hypotenuse? A. 13 B. 23 C. 37 D. 41 Ans : B 76. The angle of elevation of the top of a tower 30 m high, from two points on the level ground on its opposite sides are 45 degrees and 60 degrees. What is the distance between the two points? A. 30 B. 51.96 C. 47.32 D. 81.96 Ans : C 77. What is the value of cot 15o + cot 75o + cot 135o - cosec 30o? A. 3 B. Infinity C. 1 D. None of these Ans : C

Ratio And Proportion 78. Rs.432 is divided amongst three workers A, B and C such that 8 times A's share is equal to 12 times B's share which is equal to 6 times C's share. How much did A get? A. Rs.192 B. Rs.133 C. Rs.144 D. Rs.128 Ans : C 79. If 20 men or 24 women or 40 boys can do a job in 12 days working for 8 hours a day, how many men working with 6 women and 2 boys take to do a job four times as big working for 5 hours a day for 12 days? A. 8 men B. 12 men C. 2 men D. 24 men Ans : C 80. Two cogged wheels of which one has 32 cogs and other 54 cogs, work into each other. If the latter turns 80 times in three quarters of a minute, how often does the other turn in 8 seconds? A. 48 B. 135 C. 24 D. None of these Ans : C

Quantitative Section : Quantitative Ability Ratio And Proportion 81. The monthly incomes of A and B are in the ratio 4 : 5, their expenses are in the ratio 5 : 6. If 'A' saves Rs.25 per month and 'B' saves Rs.50 per month, what are their respective incomes? A. Rs.400 and Rs.500 B. Rs.240 and Rs.300 C. Rs.320 and Rs.400 D. Rs.440 and Rs.550

Ans : A 82. The proportion of milk and water in 3 samples is 2:1, 3:2 and 5:3. A mixture comprising of equal quantities of all 3 samples is made. The proportion of milk and water in the mixture is A. 2:1 B. 5:1 C. 99:61 D. 227:133 Ans : D 83. A group of workers can do a piece of work in 24 days. However as 7 of them were absent it took 30 days to complete the work. How many people actually worked on the job to complete it? A. 35 B. 30 C. 28 D. 42 Ans : C 84. A, B and C play cricket. A's runs are to B's runs and B's runs are to C's as 3:2. They get altogether 342 runs. How many runs did A make? A. 162 B. 108 C. 72 D. None of these Ans : A 85. The monthly salaries of two persons are in the ratio of 4:7. If each receives an increase of Rs.25 in the salary, the ratio is altered to 3: 5. Find their respective salaries. A. 120 and 210 B. 80 and 140 C. 180 and 300 D. 200 and 350 Ans : D 86. A fort has provisions for 60 days. If after 15 days 500 men strengthen them and the food lasts 40 days longer, how many men are there in the fort? A. 3500 B. 4000 C. 6000 D. None of these Ans : B

87. The ratio of marks obtained by vinod and Basu is 6:5. If the combined average of their percentage is 68.75 and their sum of the marks is 275, find the total marks for which exam was conducted. A. 150 B. 200 C. 400 D. None of these. Ans : B 88. The present ages of A and B are as 6 : 4. Five years ago their ages were in the ratio 5 : 3. Find their present ages. A. 42, 28 B. 36, 24 C. 30, 20 D. 25, 15 Ans : C 89. A, B and C enter into a partnership by investing Rs.3600, Rs.4400 and Rs.2800. A is a working partner and gets a fourth of the profit for his services and the remaining profit is divided amongst the three in the rate of their investments. What is the amount of profit that B gets if A gets a total of Rs. 8000? A. 4888.88 B. 9333.33 C. 4000 D. 3666.66 Ans : A 90. A, B and C, each of them working alone can complete a job in 6, 8 and 12 days respectively. If all three of them work together to complete a job and earn Rs.2340, what ill be C's share of the earnings? A. Rs.520 B. Rs.1080 C. Rs.1170 D. Rs.630 Ans : A

Quantitative Section : Quantitative Ability

Mixtures and Alligations 91. A 20 litre mixture of milk and water contains milk and water in the ratio 3 : 2. 10 litres of the mixture is removed and replaced with pure milk and the operation is repeated once more. At the end of the two removal and replacement, what is the ratio of milk and water in the resultant mixture? A. 17 : 3 B. 9 : 1 C. 3 : 17 D. 5 : 3 Ans : B 92. In what ratio must a person mix three kinds of tea costing Rs.60/kg, Rs.75/kg and Rs.100 /kg so that the resultant mixture when sold at Rs.96/kg yields a profit of 20%? A. 1 : 2 : 4 B. 3 : 7 : 6 C. 1 : 4 : 2 D. None of these Ans : C 93. A merchant mixes three varieties of rice costing Rs.20/kg, Rs.24/kg and Rs.30/kg and sells the mixture at a profit of 20% at Rs.30 / kg. How many kgs of the second variety will be in the mixture if 2 kgs of the third variety is there in the mixture? A. 1 kg B. 5 kgs C. 3 kgs D. 6 kgs Ans : B 94. How many litres of water should be added to a 30 litre mixture of milk and water containing milk and water in the ratio of 7 : 3 such that the resultant mixture has 40% water in it? A. 7 litres B. 10 litres C. 5 litres D. None of these Ans : C 95. How many kgs of Basmati rice costing Rs.42/kg should a shopkeeper mix with 25 kgs of ordinary rice costing Rs.24 per kg so that he makes a profit of 25% on selling the mixture at Rs.40/kg? A. 20 kgs B. 12.5 kgs C. 16 kgs D. 200 kgs

Ans : A 96. How many litres of a 12 litre mixture containing milk and water in the ratio of 2 : 3 be replaced with pure milk so that the resultant mixture contains milk and water in equal proportion? A. 4 litres B. 2 litres C. 1 litre D. 1.5 litres Ans : B 97. A sample of x litres from a container having a 60 litre mixture of milk and water containing milk and water in the ratio of 2 : 3 is replaced with pure milk so that the container will have milk and water in equal proportions. What is the value of x? A. 6 litres B. 10 litres C. 30 litres D. None of these Ans : B 98. A zookeeper counted the heads of the animals in a zoo and found it to be 80. When he counted the legs of the animals he found it to be 260. If the zoo had either pigeons or horses, how many horses were there in the zoo? A. 40 B. 30 C. 50 D. 60 Ans : C 99. From a cask of milk containing 30 litres, 6 litres are drawn out and the cask is filled up with water. If the same process is repeated a second, then a third time, what will be the number of litres of milk left in the cask? A. 0.512 liters B. 12 liters C. 14.38 liters D. 15.36 liters Ans : D 100.In a km race, A gives B a start of 20 seconds and beats him by 40m. However, when he gives B a start of 25 seconds they finish in a dead heat. What is A's speed in m/sec? A. 12.5 m/sec B. 20 m/sec C. 8 m/sec D. 10 m/sec

Ans : D

Verbal Section : Sentence Completions Directions: Each sentence below has one or two blanks. Each blank shows that something has been omitted. Under each sentence five words are given as choice. Choose the one correct word for each blank that best fits the meaning of the sentences as a whole. 1. The fact that the- of confrontation is no longer as popular as it once was procatss in race relations. A. insidiousness - reiterates B. practice - inculcates C. glimmer - foreshadows D. technique - presages E. reticence - indicates Ans :D 2. A child should not be - as being either very shy or over - agcatssive. A. categorized B. instructed C. intoned D. distracted E. refrained Ans :A 3. President Anwar el - Sadat of Egypt, disregarding - criticism in the Alab world and in his own Government, - accepted prime minister Menahem Begin's invitation to visit Israel in order to address the Israeli parliament. A. acrimonious - formally B. blemished - stiffly C. categorical - previously D. malignant - plaintively E. charismatic - meticulously Ans :A 4. In his usual - manner, he had insured himself against this type of loss. A. pensive B. providential C. indifferent D. circumspect E. caustic

Ans :D 5. We never believed that he would resort to - in order to achieve his goal; we always regarded him as a - man. A. charm - insincere B. necromancy - pietistic C. logic - honorable D. prestidigitation - articulate E. subterfuge - honest Ans :E 6. The Sociologist responded to the charge that her new theory was - by pointing out that it did not in fact contradict accepted sociological principles. A. unproven B. banal C. superficial D. complex E. heretical Ans :E 7. Despite assorted effusion to the contrary, there is no necessary link between scientific skill and humanism, and quite possibly, there may be something of a - between them. A. dichotomy B. congruity C. reciprocity D. fusion E. generosity Ans :E 8. The most technologically advanced societies have been responsible for the catatest - indeed savagery seems to be indirect proposition to A. inventions - know-how B. wars - viciousness C. triumphs - civilizations D. atrocities - development E. catastrophes - ill-will Ans :D 9. Ironically, the party leaders encountered no catater - their efforts to build as Procatssive Party than the - of the procatssive already elected to the legislature. A. obstacle to - resistance B. support for - advocacy C. praise for - reputation D. threat to - promise

E. benefit - success Ans :A 10. The simplicity of the theory - its main attraction - is also its - for only by - the assumptions of the theory is it possible to explain the most recent observations made by researchers. A. glory - rejecting B. liability - accepting C. undoing - supplementing D. downfall - considering E. virtue - qualifying Ans : C

Verbal Section : Sentence Completions 11. That the Third Battalion's fifty percent casually rate transformed its assault on Hill 306 from a brilliant stratagem into a debacle does not - eyewitness reports of its commander's extra-ordinary - in deploying his forces. A. invalidate - brutality B. gainsay - cleverness C. underscore - ineptitude D. justify - rapidity E. corroborate -determination Ans : B 12. No longer - by the belief that the world around us was expressly designed for humanity, many people try to find intellectual - for that lost certainty in astrology and in mysticism. A. satisfied - reasons B. reassured - justifications C. restricted - parallels D. sustained - substitutes E. hampered - equivalents Ans : D 13. In eighth-century Japan, people who - wasteland were rewarded with official ranks as part of an effort to overcome the shortage of - fields. A. cultivated - domestic B. located - desirable C. conserved - forested D. reclaimed - arable E. irrigated - accessible.

Ans :D 14. Clearly refuting sceptics, researchers have - not only that gravitational radiation exists but that it also does exactly what the theory- it should do. A. assumed - deducted B. estimated - accepted C. supposed - asserted D. doubted - warranted E. demonstrated - predicted. Ans :E 15. Melodramas, which presented stark oppositions between innocence and criminality, virtue and corruption, good and evil, were popular precisely because they offered the audience a world - of A. deprived - polarity B. full - circumstantiality C. bereft - theatricality D. devoid - neutrality E. composed - adversity. Ans :D 16. Sponsors of the bill were-because there was no opposition to it within the legislative, until after the measure had been signed into law. A. well-intentioned B. persistent C. detained D. unreliable E. relieved. Ans :B 17. Ecology, like economics, concerns itself with the movement of valuable through a complex network of producers and consumers. A. nutrients B. dividends C. communications D. artifacts E. commodities. Ans :C 18. Having fully embraced the belief that government by persuasion is preferable to government by - the leaders of the movement have recently - most of their previous statements supporting totalitarianism. A. proclamation - codified B. coercion - repudiated C. participation - moderated D. intimidation - issued

E. demonstration - deliberated. Ans :B 19. It would be difficult for one so - to be led to believe that all men are equal and that we must disregard race, color and creed. A. tolerant B. democratic C. broadminded D. emotional E. intolerant. Ans :E 20. Many philosophers agree that the verbal aggression of profanity in certain redical newspapers is not - or childish, but an assault on - essential to the revolutionary's purpose. A. insolent - sociability B. trivial - decorum C. belligerent - fallibility D. serious - propriety E. deliberate - affectation. Ans :B

Verbal Section : Sentence Completions 21. The - tones of the flute succeeded in - his tense nerves. A. rhapsodic - minimising B. blatant - enhancing C. hovendous - calming D. vibrant - portraying E. mellifluous - soothing. Ans :E 22. Without the psychiatrist's promise of confidentiality, trust is - and the patient's communication limited; even though confidentiality can thus be seen to be precious in thercopy, moral responsibility sometimes requires a willingness to it. A. lost - forget B. implicit - extend C. impaired - sacrifise D. ambiguous - apply E. assumed - examine. Ans :C

23. Parts of seventeenth-century Chinese pleasure gardens were not necessarily intended to look -they were designed expressly to evoke the agreeable melancholy resulting from a sense of the - of natural beauty and human glory. A. great - immutability B. joyful - mortality C. conventional - wildness D. cheerful - transitoriness E. colorful - abstractness. Ans :D 24. Despite the - of many of their colleagues, some scholars have begun to emphasize ''pop culture'' as a key for - the myths, hopes, and fears of contemporary society. A. pedantry - reinstating B. enthusiasm - symbolizing C. skepticism - deciphering D. antipathy - involving E. discernment - evaluating. Ans :C 25. If duty is the natural - of one's the course of future events, then people who are powerful have duty placed on them whether they like it or not. A. outgrowth - control over B. arbiter - responsibility for C. correlate - understanding of D. determinant - involvement in E. mitigant - preoccupation with . Ans :A 26. Clearly refuting sceptics, researches have - not only that gravitational radiation exists but that it also does exactly what the theory - it should do. A. supposed - asserted B. voubted -warranted C. assumed - deduced D. demonstrated - predicted E. estimated - accepted Ans :D 27. The Neolatonists' conception of a deity, in which perfection was measured by abundant fecundity, was contradicted by that of the Aristotelians, in which perfection was displayed in the - of creation. A. variety B. economy C. profusion D. clarity E. precision.

Ans :B 28. It is a great - to be able to transfer useful genes with as little extra gene material as possible, because the donor's genome may contain, in addition to desirable genes, many genes with - effects. A. Disappointment - superfluous B. Convenience - exquisite C. Advantage - deleterious D. Accomplishment - profound E. Misfortune - unpredictable. Ans :C 29. While admitting that the risks incurred by use of the insecticide were not - the manufacturer's spokesperson argued that effective - were simply not available. A. indeterminable - safeguards B. unusual - alternatives C. inconsequential - substitutes D. proven - antidotes E. increasing - procedures. Ans :C 30. Human reaction to the realm of though is often as strong as that to sensible presences; our higher moral life is based on the fact that - sensations actually present may have a weaker influence on our action than do ideas of - facts. A. emotional - impersonal B. familiar : symbolic C. disturbing - ordinary D. material - remote E. defenitive - controvoisial. Ans :D

Verbal Section : Sentence Completions 31. Some scientists argue that carbon compounds play such a central role in life on earth because of the possibility of - resulting from the carbon atom's ability to form an unending series of different molecules. A. variety B. stability C. deviations D. invigorations E. reproduction. Ans :A

32. It would be difficult for one so - to be led to believe that all men are equal and that we must disregard race, color and creed. A. intolerant B. democratic C. emotional D. patient E. broadminded. Ans :A 33. An occasional - remark spoiled the - that made the paper memorable. A. colloquial B. trite - cliches C. urbane - sophisticated D. hackneyed - originality E. jovial - fun. Ans :D 34. Broadway audiences have become inured to - and so - to be pleased as to make their ready ovations meaningless as an indicator of the quality of the production before them. A. cleverness : eager B. condescension : disinclined C. sentimentality : reluctant D. mediocrity : desperate E. histrionics : unlikely Ans :D 35. Nineteenth - century scholars, by examining earlier geometric Greek art, found that classical Greek art was not a magical - or a brilliant - blending Egyptian and Assyruin art, but was independently evolved by Greeks in Greece. A. conversion - annexation B. apparition - amalgam C. stratagem - appropriation D. paradigm - construct E. example - synthesis Ans :B 36. The struggle of the generations is one of the obvious constants of human affairs; therefore, it may be presumptuous to suggest that the rivalry between young and old in western society during the current decade is - critical. A. archetypally B. perennially C. disturbingly D. uniquely E. cautiously

Ans :D 37. Even though in today's Soviet union the - Muslim clergy have been accorded power and privileges, the Muslim laity and the rank - and - file clergy still. Have little - to practice their religion. A. adversaries of - inclination B. traditionalists among - incentive C. practitioners among - opportunity D. leaders of - latitude E. dissidents within -obligation Ans :D 38. Unlike the Shakespearean plays, The ''closet dramas'' of the nineteenth century were meant to be - rather than A. seen - acted B. read - acted C. produced - acted D. quiet - loud E. sophisticated - urbane Ans :B 39. The little - known but rapidly expanding use of computers in mapmaking is technologically similar to the more - uses in designing everything from bolts to satellites. A. ingenuous B. recent C. secure D. publicized E. successful Ans :D 40. Although his out numbered troops fought bravely, the general felt he had no choice but to - defeat and - a retreat. A. oversee - reject B. acknowledge - order C. hasten - suggest D. seek - try E. overcome - request Ans :B

Verbal Section: Analogies

Directions: Each of the questions below consists of two words that have a certain relationship to each other, followed by five lettered pairs of related words. Select the lettered pair of words. 1. ANGLE : DECATE A. area : square inch B. milk : quart C. society : classes D. letter : alphabet E. time : minutes Ans : A 2. CONFIRMED : INVETERATE A. knowledge : supposed B. financial : bankrupt C. immature : callow D. credible : incredible E. careful: punishing Ans :B 3. LULLABY : BARCAROLE A. birth : marriage B. night : morning C. cradle : gondola D. song : poem E. carol : sonneteer Ans :C 4. ZOOLOGY : ANIMALS A. ecology : pollution B. botany : plants C. chemistry : atoms D. history : people E. mathematics : geometry Ans :A 5. DORY : VAN A. dairy : cow B. fish : vehicle C. freighter : caisson D. runners : wheels E. danish : Dutch Ans : C 6.

PARQUET

: WOOD

A. B. C. D. E.

color : painting mosaic : glass potpourri : medley collage : tapestry linoleum : marble

Ans : B 7. SAW : CARPENTER A. Scissors : tailor B. Wagon : farmer C. Brush : painter D. Typewriter : author E. Trowel : bricklayer Ans : A 8. LURK : WAIT A. boost : elevate B. deplete : drain C. abscond : depart D. bilk : cheat E. topple : stabilize Ans : C 9. ALCHEMY : SCIENCE A. nostrum : remedy B. sideshow : carnival C. ploy : tactic D. forgery : imitation E. burlesque : comedy Ans : A 10. NEEDLE : KNIT A. bait : fish B. match : fire C. loom : weave D. soap : wash E. bed : sleep Ans : C

Verbal Section: Analogies 11. PARENTHESIS : EXPLANATION

A. B. C. D. E.

ellipsis : omission asterisk : exaggeration synopsis : affectation apostrophe : annotation synthesis : interpolation

Ans : A 12. CENSUS : POPULATION A. manifest : debts B. roster : audience C. itinerary : journeys D. inventory : merchandise E. state : incumbents Ans : D 13. STANZA : POEM A. mimicry : pantomime B. duet : chorus C. act : opera D. rhyme : verse E. pirouette : ballet Ans : C 14. EXHORT : SUGGEST A. conspire : plan B. tamper : adjust C. crave : accept D. goad : direct E. instruct : teach Ans : D 15. SAND PAPER : ABRASIVE A. gasoline : refined B. grativity : irritant C. polish : floors D. acrylic : emulsion E. oil : lubricant. Ans :E 16. DIAPHANOUS : CACOPHONOUS A. translucent : transparent B. transparent : noisy C. sheer : opaque D. harmonious : discordant E. twofold : multiple.

Ans :B 17. INFANCY : SENILITY A. january : October B. incipient : critical C. day : night D. conclusion : climax E. dawn : dusk. Ans :E 18. RIG : CONTEST A. solve : conundrum B. predict : race C. repudiate : thesis D. gerrymander : district E. incriminate : evidence Ans :D 19. ARBORETUM : TREES A. aviary : birds B. catenhouse : garden C. museum : painters D. grove : forest E. zoo : range Ans :D 20. MENDICANT : IMPECUNIOUS A. hat : askew B. liar : poor C. complainer : petulant D. critic : quizzical E. philanthrophist : prodigal. Ans :C

erbal Section: Analogies 21. RELAPSE : CONVALESCENCE A. dissonance : harmony B. feudalism : industrialization C. repetition : monotony D. impasse : debate E. recidivism : rehavbilitation. Ans :E

22. BOUQUET : FLOWERS A. corn : husk B. woodpile : logs C. forest : thicket D. mist : fog E. drift : snow. Ans :B 23. TRIANGLE : QUADRILATERAL A. rectangle : octagon B. cone : cube C. pentagon : hexagon D. plane : solid E. regular : symmetrical. Ans :C 24. SARTORIAL : TAILOR A. thespian : designer B. rhetorical : questioner C. pictorial : musician D. histrionic : singer E. terpsichorear : dancer. Ans :E 25. NECROMANCY : GHOSTS A. magic : legerdemain B. alchemy : gold C. sorcery : spirits D. fortune_telling : gypsies E. romance : stories. Ans :C 26. DRUM : TYMPANI A. piano : orchestra B. cornet : percussion C. stick : baton D. violin : viola E. oboe : woodwind. Ans :E 27. EXTROVERT : RETICENT A. reprobate : humility B. strategist : decisiveness C. zealot : loyalty D. maverick : conformity

E. renegade : ambition. Ans :D 28. HYGROMETER : BAROMETER A. snow : rain B. humidity : pressure C. water : mercury D. temperature : weather E. forecast : rain. Ans :B 29. EXEMPTION : EXCLUSIONS A. discharge : elimination B. debarment : prevention C. immunity : isolation D. forgive : condone E. enclosure : open. Ans :C 30. FEBRILE : ILLNESS A. classic : cultivation B. delusional : insanity C. eccentric : discrimination D. tenacious : astonishment E. juvenile : maturity. Ans :B

Verbal Section: Analogies 31. DISAPPROBATION : CONDEMN A. calumny : eulogise B. enigma : enlighter C. fallacy : diseminate D. exhortation : urge E. solvency : deploy. Ans :D 32. GEM : TURQUOISE A. lettuce : green B. pear : orange C. stone : magnetta D. vine : cherry

E. flower : violet. Ans :E 33. WINE : GRAPES A. liquor : intoxicating B. whiskey : hops C. champagne : raisins D. vodka : potatoes E. vineyard : winery. Ans :D 34. DEBATE : FORENSIC A. concerto : harmonizing B. drama : histrionic C. opera : spoken D. argument : domestic E. novel : original. Ans :B 35. NOISOME : GARBAGE A. heavy : metal B. warmth : snow C. fragrant : incense D. liquid : perfume E. loud : music. Ans :C 36. CONDUIT : WATER A. behaviour : liquid B. electricity : television C. artery : blood D. wire : sound E. pump : oil. Ans :C 37. BIZARRE : EXOTIC A. wild : tame B. lively : livid C. stage : dancer D. commonplace : routine E. ordinary : exceptional. Ans :D 38. ENTREPRENEUR : LABORER

A. B. C. D. E.

mediator : conflict capitalism : communism profits : wages arbitrator : capitalist moonlighting : worker.

Ans :C 39. ANTIMACASSAR : SOFA A. picture : frame B. rug : floor C. pillow : bed D. door : window E. table : chair. Ans :B 40. NOTABLE : NOTORIOUS A. heinous : atrocious B. philandering : pleasant C. philanthropic : miserly D. nefarious : secret E. philanthropic : benevolent. Ans :E CAT:Verbal : Analogies

CAT Sample Questions

Verbal Section: Analogies 41. BABBLE : TALK A. though : blank B. look : espy C. wink : eye D. leer : ogle E. simper : smile. Ans :E 42. ALCOVE : RECESS A. column : entrance B. foundation : building C. dome : roof D. turret : chimney E. foyer : ballroom

Ans :C 43. FIRM : INTRANSIGHT A. faithful : resolute B. improvident : industrious C. vague : inattentive D. concerned : obsessed E. malleable : tractable Ans :D 44. EPAULET : SHOULDER A. medal : chest B. decoration : uniform C. knapsack : back D. sword : scabbard E. sash : window Ans :A 45. ANACHRONISM : CHRONOLOGY A. tradition : custom B. variations : incongruity C. fallacy : logic D. archetype : paradigm E. debauchery : appetites Ans :C 46. DETRITUS : GLACIERS A. thaw : cold B. snow : icebergs C. sediment : bottom D. silt : rivers E. dregs : society Ans :D 47. OUTSKIRTS : TOWN A. water : goblet B. margin : page C. rung : ladder D. hangar : airplane E. trunk : tree Ans :B 48. EQUIVOCATE : COMMITMENT A. collaborate : falsification B. fabricate : explanation

C. procrastinate : action D. expostulate : confusion E. implicate : exposition Ans :C 49. MORPHINE : SEDATES A. oil : smears B. bandage : protects C. drug : addicts D. liquor : sedates E. medicine : soothes Ans :D 50. STICKLER : APPROXIMATION A. Lluggard : indolence B. connoisseur : anachronism C. scientist : theorizing D. leader : guidance E. purist : adulteration Ans :E

eading Comprehension Directions: Each reading passage in this section is followed by questions based on the content of the reading passage. Read the passage carefully and chose the best answer to each question. The questions are to be answered on the basis of what is stated or implied in the passage. 1. But man is not destined to vanish. He can be killed, but he cannot be destroyed, because his soul is deathless and his spirit is irrepressible. Therefore, though the situation seems dark in the context of the confrontation between the superpowers, the silver lining is provided by amazing phenomenon that the very nations which have spent incalculable resources and energy for the production of deadly weapons are desperately trying to find out how they might never be used. They threaten each other, intimidate each other and go to the brink, but before the total hour arrives they withdraw from the brink. 1. The main point from the author's view is that A. Man's soul and spirit can not be destroyed by superpowers. B. Man's destiny is not fully clear or visible. C. Man's soul and spirit are immortal. D. Man's safety is assured by the delicate balance of power in terms of nuclear weapons. E. Human society will survive despite the serious threat of total annihilation.

Ans : E 2. The phrase 'Go to the brink' in the passage means A. Retreating from extreme danger. B. Declare war on each other. C. Advancing to the stage of war but not engaging in it. D. Negotiate for peace. E. Commit suicide. Ans : C 3. In the author's opinion A. Huge stockpiles of destructive weapons have so far saved mankind from a catastrophe. B. Superpowers have at last realized the need for abandoning the production of lethal weapons. C. Mankind is heading towards complete destruction. D. Nations in possession of huge stockpiles of lethal weapons are trying hard to avoid actual conflict. E. There is a Silverlining over the production of deadly weapons. Ans : D 4. 'Irrepressible' in the second line means A. incompatible B. strong C. oppressive D. unrestrainable E. unspirited Ans : D 5. A suitable title for the above passage is A. Destruction of mankind is in evitable. B. Man's desire to survive inhibits use of deadly weapons. C. Mounting cost of modern weapons. D. Threats and intimidation between super powers. E. Cowardly retreat by man Ans : B

Reading Comprehension 2. Disequilibrium at the interface of water and air is a factor on which the transfer of heat and water vapor from the ocean to the air depends. The air within about a millimeter of the water is almost saturated with water vapor and the temperature of the air is close to that of the surface water. Irrespective of how small these differences might be, they are crucial, and the disequilibrium is maintained by air near the surface mixing with air higher up, which is typically appreciably cooler and lower in water

vapor content. The turbulence, which takes its energy from the wind mixes the air. As the speed of wind increases, so does the turbulence, and consequently the rate of heat and moisture transfer. We can arrive at a detailed understanding of this phenomenon after further study. The transfer of momentum from wind to water, which occurs when waves are formed is an interacting-and complicated phenomenon. When waves are made by the wind, it transfers important amounts of energy-energy, which is consequently not available for the production of turbulence. 1. This passage principally intends to: A. resolve a controversy B. attempt a description of a phenomenon C. sketch a theory D. reinforce certain research findings E. tabulate various observations Ans : B 2. The wind over the ocean usually does which of the following according to the given passage? I. Leads to cool, dry air coming in proximity with the ocean surface. II. Maintains a steady rate of heat and moisture transfer between the ocean and the air. III. Results in frequent changes in the ocean surface temperature. A. I only B. II only C. I and II only D. II and III only E. I, II, and III Ans : A 3. According to the author the present knowledge regarding heat and moisture transfer from the ocean to air as A. revolutionary B. inconsequential C. outdated D. derivative E. incomplete Ans : E 4. According to the given passage, in case the wind was to decrease until there was no wind at all, which of the following would occur? A. The air, which is closest to the ocean surface would get saturated with water vapor. B. The water would be cooler than the air closest to the ocean surface. C. There would be a decrease in the amount of moisture in the air closest to the ocean surface. D. There would be an increase in the rate of heat and moisture transfer.

E. The temperature of the air closest to the ocean and that of the air higher up would be the same. Ans : A

Reading Comprehension 3. The Food and Drug Administration has formulated certain severe restrictions regarding the use of antibiotics, which are used to promote the health and growth of meat animals. Though the different types of medicines mixed with the fodder of the animals kills many microorganisms, it also encourages the appearance of bacterial strains, which are resistant to anti-infective drugs. It has already been observed that penicillin and the tetracyclines are not as effective therapeutically as they once used to be. This resistance to drugs is chiefly caused due to tiny circlets of genes, called plasmids, which are transferable between different species of bacteria. These plasmids are also one of the two kinds of vehicles on which molecular biologists depend on while performing gene transplant experiments. Existing guidelines also forbid the use of plasmids, which bear genes for resistance to antibiotics, in the laboratories. Though congressional dabate goes on as to whether these restrictions need to be toughened with reference to scientists in their laboratories, almost no congressional attention is being paid to an ill advised agricultural practice, which produces deleterious effects. 1. In the present passage, the author's primary concern is with: A. The discovery of methods, which eliminate harmful microorganisms without generating drug-resistant bacteria. B. Attempting an explanation of the reasons for congressional inaction about the regulation of gene transplant experiments. C. Portraying a problematic agricultural practice and its serious genetic consequences D. The verification of the therapeutic ineffectiveness of anti-infective drugs E. Evaluation of the recently proposed restrictions, which are intended to promote the growth of meat animals. Ans : C 2. As inferred from the above passage, the mutual transfer of plasmids between different bacteria can result in which of the following? A. Microorganisms, which have an in-built resistance to drugs B. Therapeutically useful circlets of genes C. Penicillin like anti-infective drugs D. Viruses used by molecular biologists E. Carriers for performing gene transplant experiments. Ans : A

3. According to the above passage the author believes that those who favor the stiffening of restrictions on gene transplant research should logically also. A. Approve and aid experiments with any plasmids except those, which bear genes for antibiotic resistance. B. Inquire regarding the addition of anti-infective drugs to livestock feeds C. Oppose the using of penicillin and tetracyclines in order to kill microorganisms D. Agree to the development of meatier live-stock through the use of antibiotics E. Approve of congressional debate and discussion regarding science and health issues. Ans : B 4. The attitude the author has with reference to the development of bacterial strains that render antibiotic drugs in effective can best be described as A. indifferent B. perplexed C. pretentious D. insincere E. apprehensive Ans : E

Verbal Section : Antonyms Directions: Each of the CAT sample antonyms questions below consists of a word printed in Italics, followed by five words or phrase as choices. Choose the word or phrase which is most nearly opposite in meaning to the word in capitals and shade the alphabets marked in the grid on your answer sheet. Following are some CAT sample antonyms questions. 1. ABOMINATE : A. loathe B. despise C. adore D. abhor E. attach Ans : C 2. OBSEQUIOUS :

A. B. C. D. E.

servile first fawning supercilious improper

Ans : D 3. OROTUND : A. not resonant B. not reddish C. not eager D. pompous E. loud Ans : A 4. RECANT : A. entangle B. rescue C. fail D. assert E. predict Ans : D 5. UPBRAID : A. defer B. vacillate C. sever D. conjoin E. laud Ans : E 6. PLENITUDE : A. luxury B. magnificence C. richness D. contentment E. scarcity Ans : E 7. SCURRILOUS : A. decent B. savage C. major D. volatile E. scabby

Ans : A 8. FULMINATION : A. praise B. repetition C. escape D. ratification E. addition Ans : A 9. DISTEND A. deteriorate B. weaken C. constrict D. concentrate E. fold Ans : C 10. TOUT A. cast aspersions on B. deny the relevance of C. placate D. withhold consent E. misrepresent Ans : E

Verbal Section : Antonyms 11. SQUALID A. fervid B. florid C. pristine D. extraneous E. abundant Ans : C 12. SCOTCH A. renovate B. entrust C. unfasten D. encourage E. emphasize

Ans : D 13. PERFIDY A. tact B. generosity C. thoroughness D. loyalty E. gratitude Ans : D 14. OUTLANDISH A. conventional B. prolific C. unchanging D. transparent E. noticeable Ans : A 15. PLUMB A. reversed B. lofty C. horizontal D. thin E. light Ans : C 16. FERVID A. undistinguished B. unexpected C. stubborn D. restrained E. discouraged Ans : D 17. VACUITY A. quality B. certainty C. plenitude D. stability E. incontinence Ans : C 18. RAVEL A. knit B. omit

C. remain silent D. measure E. increase in value Ans : A 19. PERSISTENCE A. irrelevance B. inconstancy C. inequality D. intemperance E. incompetence. Ans : B 20. SUBROSA A. openly B. fashionably C. under the owse D. simply E. clandestinely Ans : A

Verbal Section : Antonyms 31. ANIMOSITY A. parody B. retardation C. sincerity D. refutation E. canaraderie Ans : E 32. INVETERATE A. uninvited B. illiterate C. cumulative D. beginning E. incompetent Ans : D 33. SCOTCH A. renovate B. encourage C. entrust D. ameliorate

Ans : B 34. PREDILECTION A. ambiguity B. unwillingness to choose C. desire to please D. propensity to dislike E. stereotype Ans : D 35. CHOLERIC A. good-natured B. spoiled C. irascible D. immune E. idiotic. Ans : A 36. EXACERBATE A. contemplate B. bewilder C. reward D. better E. horify Ans : D 37. EQUANIMITY A. clamour B. volume C. disparity D. agitation E. caution Ans : D 38. ANIMADVERSION A. gullibility B. precision C. praise D. sobriety E. criticize Ans : C 39. EXHUME A. enter B. fertilize

C. inter D. decay E. clarify Ans : C 40. CALLOW A. rustic B. crude C. exquisite D. experienced E. migratory Ans : D

Verbal Section : Antonyms 21. PREFATORY : A. intelligent B. outstanding C. predatory D. conclusive E. magnificent Ans : E 22. CONCILIATE : A. arrive B. appeal C. retaliate D. estrange E. lie Ans : B 23. SUBSERVIENT : A. fawning B. obsequious C. miserly D. omnipresent E. haughty Ans : C 24. VAUNTED : A. berated B. belittled

C. lauded D. wicked E. worried Ans : C 25. QUOTA : A. Anonymous remark B. decisive action C. debatable issue D. unlimited number E. irrelevant topic Ans : D 26. CONTENTIOUS : A. satisfied B. pacific C. hungry D. bellicose E. dissatisfied Ans : D 27. OBLOQUY : A. fame B. name C. inquiry D. shame E. collogue Ans : D 28. PENCHANT : A. distaste B. scabbard C. agreement D. earring E. beginning Ans : C 29. BALEFUL : A. empty B. tasty C. gaudy D. full E. congenial Ans : D

30. CURT : A. contractual B. precise C. honest D. voluble E. peremptory Ans : D

Verbal Section : Antonyms 41. CUPIDITY A. generosity B. love C. anxiety D. entertainment E. tragedy. Ans : A 42. ANIMOSITY A. parody B. retardation C. sincerity D. refutation E. canaraderie Ans : B 43. INVETERATE A. uninvited B. illiterate C. cumulative D. beginning E. incompetent Ans : A 44. SATURNINE : A. quick – wilted B. genial C. heavy – handed D. distinguished E. devout Ans : E 45. PERSPICACIOUS : A. Insufficiently precise

B. C. D. E.

of indefinite duration dull wilted lacking intrinsic value condemnatory

Ans : D 46. INCARCERATE : A. summon B. assist C. liberate D. anticipate E. confide Ans : C 47. INSOLVENCY : A. ability to sustain growth B. concentration C. coherence D. ability to pay one’s debts E. compatibility Ans : D 48. EFFLUVIA : A. controlled reactions B. predictable results C. important examples D. descried products E. relevant theories Ans : C 49. APPOSITE : A. parallel B. synonymous C. hostile D. inappropriate E. vague Ans : D 50. GRATUITOUS : A. frank B. pithy C. warranted D. frugal E. ingenuous

Ans : A

Verbal Section : Antonyms 51. PREFATORY : A. intelligent B. outstanding C. predatory D. conclusive E. magnificent Ans : E 52. CONCILIATE : A. arrive B. appeal C. retaliate D. estrange E. lie Ans : B 53. SUBSERVIENT : A. fawning B. obsequious C. miserly D. omnipresent E. haughty Ans : C 54. VAUNTED : A. berated B. belittled C. lauded D. wicked E. worried Ans : C 55. QUOTA : A. Anonymous remark B. decisive action C. debatable issue D. unlimited number E. irrelevant topic

Ans : D 56. CONTENTIOUS : A. satisfied B. pacific C. hungry D. bellicose E. dissatisfied Ans : D 57. OBLOQUY : A. fame B. name C. inquiry D. shame E. collogue Ans : D 58. PENCHANT : A. distaste B. scabbard C. agreement D. earring E. beginning Ans : C 59. BALEFUL : A. empty B. tasty C. gaudy D. full E. congenial Ans : D 60. CURT : A. contractual B. precise C. honest D. voluble E. peremptory Ans : D

Verbal Section : Antonyms

61. INVIDIOUS : A. candid B. stubborn C. defensive D. hostile E. inoffensive Ans : E 62. MACERATE : A. cover by painting B. assess by observing C. harden by drying D. influence by lying E. cure by medicating Ans : B 63. SKEPTICISM : A. reason B. conviction C. plausibility D. audricty E. argument Ans : D 64. IGNOMINIOUS : A. scholarly B. incognito C. laudatory D. disgraceful E. erudite Ans : B 65. CODA : A. creflain B. crescendo C. prelude D. improvisation E. solo Ans : A 66. PALTRY : A. farm B. scanty C. excessive D. friendly

E. benevolent Ans : A 67. PUISSANCE : A. strength B. knowledge C. liberality D. skepticism E. powerlessness Ans : E 68. MANUMIT : A. print B. impress C. enslave D. fail E. endeavor Ans : D 69. GENUFLECT : A. pronounce correctly B. falsify C. trick D. stand erect E. project Ans : E 70. INNOCUOUS : A. toxic B. large C. sober D. impeccable E. spotless Ans : C

erbal Section : Antonyms 71. BAROQUE : A. rococo B. simple C. common D. stupid

E. boat like Ans : B 72. MYOPIC : A. blind B. moral C. visionary D. farsighted E. glassy Ans : C 73. NASCENT : A. loyal B. fading C. unnamed D. treacherous E. reoccuring Ans : C 74. LOLL : A. describe exactly B. insist strongly C. comply readily D. notice incidentally E. move vigorously Ans : E 75. TURBULENCE : A. immunity B. tranquility C. meditation D. moderation E. co – ordination Ans : E 76. BANAL : A. inclined B. faithful C. elaborate D. forced E. arresting Ans : C 77. GERMINAL

A. B. C. D. E.

sterilized strategic fully developed primitive excused .

Ans : D 78. GASCONADE A. modesty B. transparency C. seizure D. cleanliness E. imposture Ans : A 79. MIASMA A. scenario B. summing up C. noxious fumes D. fragrant aroma E. benevolent Ans : B 80. OPPORTUNIST A. Man of principle B. fatalist C. fledgling D. colleague E. foe. Ans : D

Verbal Section : Antonyms 81. CENSURE A. uncertainity B. encomium C. criticism D. legal contual E. matrimony Ans : B 82. COMMODIOUS

A. B. C. D. E.

product space cramped company roomy.

Ans : E 83. EFFRONTERY A. modesty B. confrontation C. avoidance D. shamelessness E. impudence Ans : B 84. OBSTREPEROUS A. noisy B. defiant C. permeable D. quiet E. stubborn Ans : E 85. PACIFY A. ameliorate B. patchup C. truce D. tormented E. agitated Ans : C 86. AMBIGUOUS A. confusing B. lucid C. desirous D. obfuscate E. pun Ans : A 87. MILITANT A. Dramatic B. combative C. religious D. pacific E. quaint.

Ans : D 88. MOTILITY : A. static B. tension C. ascent D. liquidity E. vulnerability Ans : A 89. SINUOUS : A. wet B. vacant C. numerous D. direct E. round Ans : D 90. PLUMB : A. reversed B. horizontal C. light D. lofty E. thin Ans : B

Verbal Section : Antonyms 91. SEGMENT: A. inflate B. affix C. keep still D. make whole E. cleanse Ans : D 92. OSSIFY : A. reassemble fragments B. overlook problems C. create consensus D. placate critics E. transcend conventions

Ans : E 93. RAVEL : A. increase in value B. omit C. remain silent D. measure E. knit Ans : E 94. CALUMINATE : A. vindicate B. supplant C. rejuvenate D. follow E. familiarize Ans : A 95. TURPITUDE : A. pragmatism B. probity C. judiciousness D. animation E. determinedness Ans : B 96. INVECTIVE : A. willing compliance B. normality C. restoration D. fertility E. laudatory words Ans : E 97. PILLORY : A. lament B. foster C. exalt D. enjoy E. forgive Ans : C 98. UNTOWARD : A. experienced B. inevitable

C. industrious D. straight forward E. favourable Ans : E

Analytical Section : Analytical Reasoning Directions :All CAT sample analytical resoning questions are based on a passage or set of conditions. While answering a few of the questions, you would find it useful to draw a rough diagram. To answer any CAT sample analytical reasoning question choose the answer you think is most appropriate among the given options. Questions 1- 3 Three men (Tom, Peter and Jack) and three women (Eliza, Anne and Karen) are spending a few months at a hillside. They are to stay in a row of nine cottages, each one living in his or her own cottage. There are no others staying in the same row of houses. 1. Anne, Tom and Jack do not want to stay in any cottage, which is at the end of the row. 2. Eliza and Anne are unwilling to stay besides any occupied cottage.. 3. Karen is next to Peter and Jack. 4. Between Anne and Jack's cottage there is just one vacant house. 5. None of the girls occupy adjacent cottages. 6. The house occupied by Tom is next to an end cottage. 1. Which of the above statements can be said to have been derived from two other statements ? A. Statement 1 B. Statement 2 C. Statement 3 D. Statement 5 E. Statement 6 Ans : D 2. How many of them occupy cottages next to a vacant cottage ? A. 2 B. 3 C. 4 D. 5 E. 6 Ans : C 3. Which among these statement(s) are true ?

I.Anne is between Eliza and Jack. II.At the most four persons can have occupied cottages on either side of them. . III.Tom stays besides Peter. A.I only B.II only C.I and III only D.II and III only E.I, II and III Ans : C Questions 4 - 7 An employee has been assigned the task of allotting offices to six of the staff members. The offices are numbered 1 - 6. The offices are arranged in a row and they are separated from each other by six foot high dividers. Hence voices, sounds and cigarette smoke flow easily from one office to another. Miss Robert's needs to use the telephone quite often throughout the day. Mr. Mike and Mr. Brown need adjacent offices as they need to consult each other often while working. Miss. Hardy, is a senior employee and has to be allotted the office number 5, having the biggest window. . Mr. Donald requires silence in the offices next to his. Mr. Tim, Mr. Mike and Mr. Donald are all smokers. Miss Hardy finds tobacco smoke allergic and consecutively the offices next to hers to be occupied by non-smokers. Unless specifically stated all the employees maintain an atmosphere of silence during office hours. 4. The ideal candidate to occupy the office furthest from Mr. Brown would be A. Miss Hardy B. Mr. Mike C. Mr. Tim D. Mr. Donald E. Mr. Robert Ans : D 5. The three employees who are smokers should be seated in the offices. A. 1, 2 and 4 B. 2, 3 and 6 C. 1, 2 and 3 D. 1, 2 and 3 E. 1, 2 and 6 Ans : D 6. The ideal office for Mr. Mike would be.

A. B. C. D. E.

2 6 1 3 4

Ans : D 7. In the event of what occurrence, within a period of one month since the assignment of the offices, would a request for a change in office be put forth by one or more employees ? A. Mr. Donald quitting smoking. B. The installation of a noisy teletype machine by Miss Hardy in her office. C. Mr. Robert's needing silence in the office (s) next to her own. . D. Mr. Brown suffering from laryngitis. E. Mr. Tim taking over the duties formerly taken care of by Miss. Robert. . Ans : E

Analytical Section : Analytical Reasoning Questions 8 - 10 In an experiment conducted at a laboratory, 160 white mice were injected with Serum D. 160 other white mice were injected with a harmless sugar solution .In two weeks time 39% of the white mice, who were injected with Serum D contracted the highly contagious and often fatal disease, jungle fever. Hence, it can be concluded that jungle fever is caused by some elements similar to the elements in Serum D. 8. The above discussion would be weakened most severely in case it is shown that A. People contracting jungle fever are usually the victims of the bite of the South American Lesser Hooded Viper. B. One among the 160 white mice had already contracted jungle fever prior to the laboratory experiment. C. The natural habitats of white mice does not contain any of the elements found in Serum D. D. The scientists administered the injections being ignorant of the contents of the solutions used. E. The 160 white mice used in the laboratory experiment were kept isolated from each other. Ans : B 9. The above argument would be highly empowered in case it were shown that: A. Some of the elements in Serum D are extracted from the root of a certain poisonous jungle wildflower. B. Within a period of two weeks about 40% of the white mice, who were injected with a harmless sugar solution also contracted jungle fever.

C. Almost all the white mice died within a period of two days after the first symptoms appeared. D. Normally the rate of jungle fever among white mice is less than 0.01%. E. Invariably the blood of the victims of jungle fever victims contains a high level of a certain toxic substance also found in serum D. Ans : E 10. Distribution of leaflets and delivering speeches on government property should be outlawed. Radicals and fanatics have no right to use public property when peddling their unsavory views. The argument above is based on the postulate A. The general public has a special concern in the free exchange of different political views. B. Radicals and fanatics prefer the use of public property while propagating their viewpoint. C. Every person who hands out leaflets and delivers speeches is a radical or fanatic. D. Legal constraints which are applicable to one group need not be equally applicable to all. E. Any political activity, which hinders the proper functioning of the government should not be protected by the law. Ans : C Questions 11 - 12 Successfully adjusting to one's environment leads to happiness. War at a universal level war destroys the weaker people, who are the most unable to adjust to their environment. Thus, war at the universal level puts weaklings out of their misery and allows more space for their predators to enjoy life in a better manner. As those actions have to be performed, which maximize the level of happiness of the greatest number, war at a universal level should take place. 11. What response would the author of the above discussion come up with, in the case of the objection that the weaklings far exceed strong people? I. He would respond with the statement that the person making the objection is a weakling. II. He would respond by saying that weaklings will be miserable no matter what happens. III. He would respond with the statement that the strong would be frustrated if the weaklings are destroyed. A. B. C. D. E.

I only II only III only I and II only II and III only

Ans : E 12. The author's discussion would be greatly if he agreed to which of the following? I.Technology could change the environment. II.War at the universal level would be an integral part of the environment. III.It is possible for the strong to survive without suppressing the weak. A.I only B.II only C.III only D.I and III only E.I, II and III only Ans : A

Analytical Section : Analytical Reasoning Questions 13 - 15 Come back with us to the real America leaving behind the turmoil of civilization. The real America is still inhabited by the eagle, the buffalo, the mountain lion and elk; it is still spacious, sprawling and majestic. Experience the freedom and serenity still to be found in 13. Choose the best option to complete the above statement: A. the natural beauty of our land B. the fascinating urban centers C. the wild terrain of Africa D. one's own subconscious E. the great sprawling cities of the Southwest Ans : A 14. The above paragraph is most likely to appear in which of the following? A. A Hunter's Guide to The United States B. Exploring the Great Outdoors C. The Quiet Beauty of Alaska D. How the Eagle Became Extinct E. Returning to America Ans : D 15. When I am elected, I will work towards effecting those changes for which I have been fighting all these years. We will work together to do away with the bureaucratic bogs which have existed ever since my opponent took office. Everyone of you knows what I stand for; I invite my opponent to ... For completion of the above statement choose the best option:

A. B. C. D. E.

hand in his resignation graciously make his stance clear stop lying to the public get our city more federal aid extend his support to me

Ans : B 16. We can never make our beliefs regarding the world certain. Even scientific theory of a most rigorous and well-confirmed nature is likely to change over a decade or even tomorrow. If we refuse to even try to understand, then it is like resigning from the human race. Undoubtedly life of an unexamined kind is worth living in other respects--as it is no mean thing to be a vegetable or an animal. It is also true that a man wishes to see this speculative domain beyond his next dinner. From the above passage it is clear that the author believes that A. B. C. D. E.

men would not do well to speculate progress in the scientific field is impossible one should live life with the dictum 'what will be will be' men should ignore their animal needs men are different from animals as far as their reasoning abilities are concerned.

Ans : E Questions 17-21 essences out of a stock of five essences-- L, M, N, O, and P are used in making all perfumes by a manufacturer. He has learned that for a blend of essences to be agreeable it should comply with all the rules listed below. Two or more

A perfume containing L, should also contain the essence N, and the quantity of N should be twice as that of L. A perfume containing M, must also have O as one of its components and they should be in equal proportion. A single perfume should never contain N as well as O. O and P should not be used together. A perfume containing the essence P should contain P in such a proportion that the total amount of P present should be greater than the total amount of the other essence or essences used. 17. Among the following which is an agreeable formula for a perfume? A. One part L, one part P B. Two parts M, two parts L C. Three parts N, three parts L D. Four parts O, four parts M E. Five parts P, five parts M Ans : D

18. Adding more amount of essence N will make which of the following perfumes agreeable? A. One part L, one part N, five parts P B. Two parts M, two parts N, two parts P C. One part M, one part N, one part P D. Two parts M, one part N, four parts P E. Two parts N, one part O, three parts P Ans : A 19. Among the following, the addition of which combination would make an unagreeable perfume containing two parts N and one part P agreeable? (A) One part L (B) One part M (C) Two parts N (D) One part O (E) Two parts P Ans : E 20. Among the following which combination cannot be used together in an agreeable perfume containing two or more essences? A. L and M B. L and N C. L and P D. M and O E. P and N Ans : A 21. Among the below mentioned formulas, which can be made agreeable by the eliminating some or all of one essence ? A. One part L, one part M, one part N, four parts P B. One part L, two parts N, one part O, four parts P C. One part L, one part M, one part O, one part P D. Two parts L, two parts N, one part O, two parts P E. Two parts M, one part N, two parts O, three parts P Ans : B

Analytical Section : Analytical Reasoning 22. Everything that a person does, which is dictated by reason of ignorance is not voluntary. Involuntary actions are those which produce pain and repentance. Incase a man has done something in his ignorance and he does not feel vexed due to his action, he has not acted voluntarily as he was not aware of what he was doing, nor yet involuntarily since he is not pained. After reading this passage we can arrive at the conclusion that:

A. A person is not a voluntary agent, if he acts by reason of ignorance and repents. . B. If an action is done by reason of ignorance and is not voluntary , then it was repented. C. A man is an involuntary agent, if he acts by reason of ignorance. D. Some actions are either involuntary or not voluntary. E. If a man is not a voluntary agent, then he acted by reason of ignorance and repents. Ans : A 23. Everything that God knows necessarily is, because even what we ourselves know necessarily is; and, of course, our knowledge is not as certain as God's knowledge is. But no future contingent thing necessarily is.. Among the following statements, which naturally follows from the above: 1.There are no future contingent things. 2.It is not true that God has knowledge of only necessary things. 3.God has knowledge of no contingent future things. 4.It is not possible for us to know God. 5.God has knowledge of everything. . Ans : C Questions 24 - 25 Some lawyers are of the view that the observation of the intrinsic qualities of pornography in any composition depends on literary criticism and hence it is a matter of opinion. It is rather odd, though, that in a legal connection, serious critics themselves quite often behave as if they believed criticism to be a matter of opinion. Why be a critic - and teach in universities - in case criticism involves nothing but uttering capricious and arbitrary opinions ? 24. In the above argument the author is trying to establish that A. whether a composition can be called pornographic or not is a matter of opinion. . B. it is not a matter of opinion whether a work is pornographic. C. observance of the qualities of pornography is not dependent on literary criticism. D. critics seem hypocritical. E. critics should not teach at universities. Ans : D 25. The above discussion would be weakened if it is pointed out that: A. literary critics are of the opinion that nothing is pornographic. B. lawyers believe that the observance of the qualities of pornography is a matter of opinion, as literary critics are not in agreement in this regard. C. literary critics are not legal authorities.

D. literary critics should not concern themselves with deciding what is pornographic. E. literary critics in the teaching profession at the university level are init only for the money. Ans : B

Analytical Section : Analytical Reasoning Questions 26 - 31 Nine individuals - Z, Y, X, W, V, U, T, S and R - are the only candidates, who can serve on three committees-- A, B and C, and each candidate should serve on exactly one of the committees. Committee A should

consist of exactly one member more than committee B. It is possible that there are no members of committee C. Among Z, Y and X none can serve on committee A. Among W, V and U none can serve on committee G. Among T, S and R none can serve on committee C. 26. In case T and Z are the individuals serving on committee B, how many of the nine individuals should serve on committee C? A. 3 B. 4 C. 5 D. 6 E. 7 Ans : B 27. Of the nine individuals, the largest number that can serve together on committee C is A. 9 B. 8 C. 7 D. 6 E. 5 Ans : D 28. In case R is the only individual serving on committee B, which among the following should serve on committee A? A. W and S B. V and U C. V and T D. U and S

E. T and S Ans : E 29. In case any of the nine individuals serves on committee C, which among the following should be the candidate to serve on committee A? A. Z B. Y C. W D. T E. S Ans : C 30. In case T, S and X are the only individuals serving on committee B, the total membership of committee C should be: A. Z and Y B. Z and W C. Y and V D. Y and U E. X and V Ans : A 31. Among the following combinations which could constitute the membership of committee C? A. Y and T B. X and U C. Y, X and W D. W, V and U E. Z, X, U and R Ans : B

Analytical Section : Logical Reasoning Directions : Each CAT sample logical reasoning question in this part of the assessment starts with a reading passage containing the information to be used to choose between correct and incorrect logical conclusions. These conclusions are based on the information in the passage. After this reading passage, you are given a lead-in phrase that tells you to choose from among five different responses. These possible responses are generated by correctly or incorrectly applying logical thought to the information in the passage at the beginning of the question. They can be thought of as different ways of completing a sentence that begins with the lead-in phrase. Each reading passage is based on actual Bureau of Labor Statistics documents but is not necessarily a completely accurate representation of BLS work. It is important that you

accept every fact in the reading passage as true, when you evaluate the response choices offered. You should use only the information in the passage as the basis for accepting or rejecting any response choices. Be careful not to allow any "facts" that are not clearly stated in the reading passage, or any outside knowledge you may have of the "facts", to influence your thinking. Following are some CAT sample logical reasoning questions. 1. Testifying before the Senate committee investigating charges that cigarette manufacturers had manipulated nicotine levels in cigarettes in order to addict consumers to their products, tobacco executives argued that cigarette smoking is not addictive. The primary reason they gave in support of this claim was that cigarette smoking was not regulated by the Federal Drug Administration. For the tobacco executives' argument to be logically correct, which of the following must be assumed? A. Substances that are not addictive are not regulated by ...........the Federal Drug Administration. B. The tobacco executives lied when they claimed that ...........cigarette smoking was not addictive. C. Some addictive substances are not regulated by the ...........Federal Drug Administration. D. There is no scientific proof that cigarette smoking is ...........addictive. E. Substances that are not regulated by the Federal Drug ...........Administration are not addictive. Ans : E 2. People should be held accountable for their own behavior, and if holding people accountable for their own behavior entails capital punishment, then so be it. However, no person should be held accountable for behavior over which he or she had no control. Which of the following is the most logical conclusion of the argument above? A. People should not be held accountable for the ...........behavior of other people. B. People have control over their own behavior. C. People cannot control the behavior of other people. D. Behavior that cannot be controlled should not be ...........punished. E. People have control over behavior that is subject ...........to capital punishment. Ans : B 3. There is clear evidence that the mandated use of safety seats by children under age four has resulted in fewer child fatalities over the past five years. Compared to the five-year period prior to the passage of laws requiring the use of safety seats, fatalities of children under age four have decreased by 30 percent.

Which one of the following, if true, most substantially strengthens the argument above? A. The number of serious automobile accidents involving ...........children under age four has remained steady over the ...........past five years. B. Automobile accidents involving children have decreased ...........sharply over the past five years. C. The use of air bags in automobiles has increased by ...........30 percent over the past five years. D. Most fatal automobile accidents involving children under ...........age four occur in the driveway of their home. E. The number of teenage drivers has increased by 30 ...........percent over the past five years. Ans : A 4. Lycopene, glutathione, and glutamine are powerful antioxidants that neutralize the free radicals that are produced in the body as a result of routine bodily processes. An excess of these free radicals in your system causes rapid aging because they accelerate the rate of cellular damage. Aging is simply the result of this damage. Thus, to slow down aging it is necessary to supplement your diet with these antioxidants on a daily basis. Which of the following, if true, most seriously undermines the author's contention? A. Most persons aren't concerned with the effects of aging ...........until it is too late to do anything. B. Exercise associated with normal daily activities effectively ...........neutralizes and dissipates the free radicals that are ...........produced as a result of routine bodily processes. C. The cost of antioxidants is exorbitantly high and well ...........beyond the budget of most consumers. D. Only overweight people who do not exercise on a daily ...........basis are likely to have an excess of free radicals in their ...........systems. E. Smoking cigarettes is one of the main causes of cellular ...........damage in humans. Ans : B 5. Is it wrong for doctors to lie about their patients' illnesses? Aren't doctors just like any other people we hire to do a job for us? Surely, we would not tolerate not being told the truth about the condition of our automobile from the mechanic we hired to fix it, or the condition of our roof from the carpenter we employed to repair it. Just as these workers would be guilty of violating their good faith contracts with us if they were to do this, doctors who lie to their patients about their illnesses violate these contracts as well, and this is clearly wrong. The conclusion of the argument is best expressed by which of the following?

A. Doctors who lie to their patients about their illnesses ...........violate their good faith contracts with their patients. B. Doctors often lie to their patients about their illnesses. C. Doctors are just hired workers like mechanics and ...........carpenters. D. It is wrong for doctors to lie about their patients' ...........illnesses. E. Doctors, like mechanics and carpenters, enter into good ...........faith contracts with us when we hire them. Ans : D 6. As any economist knows, healthy people pose less of an economic burden to society than unhealthy people. Not surprisingly, then, every dollar our state government spends on prenatal care for undocumented immigrants will save taxpayers of this state three dollars. Which of the following, if true, would best explain why the statistics cited above are not surprising? A. The state's taxpayers pay for prenatal care of all ...........immigrants. B. Babies born in this state to undocumented immigrant ...........parents are entitled to infant care benefits from ...........the state. C. State benefits for prenatal care serve to promote ...........undocumented immigration. D. Babies whose mothers did not receive prenatal care ...........are just as healthy as other babies. E. Pregnant women who do not receive prenatal care are ...........more likely to experience health problems than ...........other pregnant women. Ans : E 7. Beautiful beaches attract people, no doubt about it. Just look at this city's beautiful beaches, which are among the most overcrowded beaches in the state. Which of the following exhibits a pattern of reasoning most similar to the one exhibited in the argument above? A. B. C. D. E.

usually appear at the same drinking ...........hole at the same time of day. Therefore, moose and ...........bear must grow thirsty at about the same time. Children who are scolded severely tend to misbehave ...........more often than other children. Hence if a child is ...........not scolded severely that child is less likely to ...........misbehave. This software program helps increase the work ...........efficiency of its users. As a result, these users ...........have more free time for other activities. During warm weather my dog suffers from fleas ...........more so than during cooler weather. Therefore, ...........fleas must thrive in a warm environment. Pesticides are known to cause anemia in some people. ...........However, most anemic people live in regions where ...........pesticides are not commonly used. Moose and bear

Ans : D 8. Our school district should not spend its money on the new Verbal Advantage reading program. After all, our students get all the reading practice they need by studying history and science. The argument above depends on which the following assumptions? A. The Verbal Advantage program would not help the ...........students learn history and science. B. Other reading programs are just as effective but ...........less expensive than the Verbal Advantage program. C. The Verbal Advantage program involves only reading ...........practice. D. Teaching students history and science is more ...........important than teaching them reading skills. E. The students can already read well enough to ...........study history and science Ans : C 9. A study of native born residents in Newland found that two-thirds of the children developed considerable levels of nearsightedness after starting school, while their illiterate parents and grandparents, who had no opportunity for formal schooling, showed no signs of this disability. If the above statements are true, which of the following conclusions is most strongly supported by them? A. Only people who have the opportunity for formal ...........schooling develop nearsightedness. B. People who are illiterate do not suffer from ...........nearsightedness. C. The nearsightedness in the children is caused by the ...........visual stress required by reading and other class work. D. Only literate people are nearsighted. E. One-third of the children are illiterate. Ans : C 10. Newspaper publishers earn their profits primarily from advertising revenue, and potential advertisers are more likely to advertise in newspapers with a wide circulation—a large number of subscribers and other readers—than with other newspapers. But the circulation of the newspaper that is currently the most profitable one in this city has steadily declined during the last two years, while the circulation of one of its competitors has steadily increased. Any of the following, if true, would help explain the apparent discrepancy between the two statements above EXCEPT:

A. Advertisers generally switch from the most widely ...........circulated newspaper to another one only when the ...........other one becomes the most widely circulated ...........newspaper instead. B. Advertising rates charged by the most profitable ...........newspaper in the city are significantly higher than ...........those charged by its competitors. C. The most profitable newspaper in the city receives ...........revenue from its subscribers as well from advertisers. D. The circulation of the most profitable newspaper ...........in the city is still greater than than of any of its ...........competitors. E. The number of newspapers competing viably with the ...........most profitable newspaper in the city has increased ...........during the last two years. Ans : E

Analytical Section : Logical Reasoning 11. Although most of the fastest growing jobs in today's economy will require a college degree, many of the new jobs being created-from home health aide to desktop publisherrequire knowledge other than that gained from earning a degree. For workers in those jobs, good basic skills in reading, communication, and mathematics play an important role in getting a job and developing a career. From the information given above it can be validly concluded that, in today's economy, A. skills in reading, communication, and mathematics play an important role in developing a career as a desktop publisher B. the majority of the new jobs being created require knowledge other than that gained from earning a college degree C. a job as a home health aide will rely more on communication skills than on basic skills in reading and mathematics D. if a job is one of the fastest growing jobs, it will require a college degree E. desktop publisher jobs and home health aide jobs are not among the fastest growing jobs Ans : A 12. According to the National Agricultural Aviation Society (NAAS), without the use of crop protection products to control insects, weeds, and diseases, crop yields per acre will drop by more than 50 percent. The first aerial application of insecticide occurred in 1921, and it was a huge success. By contrast, in today's economy all aircraft that are classified as aerial applicators do more than just apply insecticide; today, they also spread seed and apply fertilizer. From the information given above it CANNOT be validly concluded that A. in today's economy, if an aerial applicator is used, then it will be able to spread seed and to apply fertilizer

B. according to the NAAS, if crop yields per acre never drop by more than 50 percent, then crop protection products have been used to control insects, weeds, and diseases C. in today's economy, any aircraft that cannot be used to apply fertilizer cannot be classified as an aerial applicator D. in 1921, if an aircraft was used for the application of insecticide, then it was not also used to spread seed E. according to the NAAS, if crop yields per acre drop by more than 50 percent, then crop protection products have not been used to control insects, weeds, and diseases. Ans : E 13. No national productivity measures are available for underground industries that may exist but remain unreported. On the other hand, at least some industries that are run entirely by self-employed industrialists are included in national productivity measures. From the information given above, it can be validly concluded that A. there are at least some industries run entirely by self-employed industrialists that are underground industries B. no industries that are run entirely by self-employed industrialists operate underground C. there are at least some industries other than those run entirely by selfemployed industrialists that are underground industries D. there are at least some industries run entirely by self-employed industrialists that are not underground industries E. there are at least some underground industries for which national productivity measures are available Ans : D 14. Lou observes that if flight 409 is canceled, then the manager could not possibly arrive in time for the meeting. But the flight was not canceled. Therefore, Lou concludes, the manager will certainly be on time. Evelyn replies that even if Lou's premises are true, his argument is fallacious. And therefore, she adds, the manager will not arrive on time after all. Which of the following is the strongest thing that we can properly say about this discussion? A. Evelyn is mistaken in thinking Lou's argument to be fallacious, and so her own conclusion is unwarranted. B. Evelyn is right about Lou's argument, but nevertheless her own conclusion is unwarranted. C. Since Evelyn is right about Lou's argument, her own conclusion is well supported. D. Since Evelyn is mistaken about Lou's argument, her own conclusion must be false.

E. Evelyn is right about Lou's argument, but nevertheless her own conclusion is false. Ans : B 15. Sally has never received a violation from the Federal Aviation Administration during her 16-year flying career. Sally must be a great pilot. Which of the following can be said about the reasoning above? A. B. C. D. E.

The definitions of the terms create ambiguity. The argument uses circular reasoning. The argument works by analogy. The argument is built upon hidden assumptions. This is an example of an argument that is directed against the source of the claim rather than the claim itself.

Ans : D 16. The Japanese economic model created strong domestic industries through subsidies from its Ministry of Trade and by closing off competitive foreign firms to its domestic market. This strategy promised to help economic growth by incubating domestic industries. New Japanese industries could count on a known local demand and would be protected from competition by tariffs and other barriers. The program could reduce the amount of imports and therefore improve the nation's balance of trade. Which of the following, based on the passage above, is a weakness in this economic strategy? A. A protectionist policy will create animosity among other nations. B. Fast growth of small industries will create a class of millionaires and increase the inequality of income. C. Subsidies and import constraints keep domestic prices high and impose a burden on consumers. D. Quotas are more regressive than tariffs. E. The demand for the products made by the incubated industries would not be known. Ans : C 17. Historically, famines have generally been followed by periods of rising wages, because when a labor force is diminished, workers are more valuable in accordance with the law of supply and demand. The Irish potato famine of the 1840s is an exception; it resulted in the death or emigration of half of Ireland's population, but there was no significant rise in the average wages in Ireland in the following decade. Which one of the following, if true, would LEAST contribute to an explanation of the exception to the generalization?

A. Improved medical care reduced the mortality rate among able-bodied adults in the decade following the famine to below prefamine levels. B. Eviction policies of the landowners in Ireland were designed to force emigration of the elderly and infirm, who could not work, and to retain a high percentage of able-bodied workers. C. Advances in technology increased the efficiency of industry and agriculture, and so allowed maintenance of economic output with less demand for labor. D. The birth rate increased during the decade following the famine, and this compensated for much of the loss of population that was due to the famine. E. England, which had political control of Ireland, legislated artificially low wages to provide English-owned industry and agriculture in Ireland with cheap labor. Ans : D 18. Cars are safer than planes. Fifty percent of plane accidents result in death, while only one percent of car accidents result in death. Which of the following, if true, would most seriously weaken the argument above? A. Planes are inspected more often than cars. B. The number of car accidents is several hundred thousand times higher than the number of plane accidents. C. Pilots never fly under the influence of alcohol, while car drivers often do. D. Plane accidents are usually the fault of air traffic controllers, not pilots. E. Planes carry more passengers than cars do. Ans : B 19. The body of anyone infected by virus X will, after a week, produce antibodies to fight the virus; the antibodies will increase in number for the next year or so. There is now a test that reliably indicates how many antibodies are present in a person's body. If positive, this test can be used during the first year of infection to estimate to within a month how long that person has had the virus. Which one of the following conclusions is best supported by the statements above? A. Antibodies increase in number only until they have defeated the virus. B. Without the test for antibodies, there is no way of establishing whether a person has virus X. C. Antibodies are produced only for viral infections that cannot be fought by any other body defenses. D. If a person remains infected by virus X indefinitely, there is no limit to the number of antibodies that can be present in the person's body. E. Anyone infected by virus X will for a time fail to exhibit infection if tested by the antibody test. Ans : E

20. Ever since I arrived at the college last week, I've been shocked by the poor behavior of the students. The student population is completely lacking in proper social skills. Which of the following, if true, would weaken the above conclusion? A. B. C. D. E.

Students who are away from their parents often exhibit rude behavior. The college numbers over 50,000 students. The narrator is a student and has interacted with many students. Social skills should not be expected of college students. The narrator was reluctant to stay at the college.

Ans : B

Analytical Section : Logical Reasoning 21. A study of a math program implemented in several pre-schools indicates that children who received the specialized Math Plus math education between the ages three and five had significantly higher math scores in 3rd and 4th grade than their classmates who did not receive this instruction. The proponents of the math argue that the introduction of this program for all children age three to five will significantly improve their chances for success in school. Which of the following, if true, would most seriously weaken the above argument? A. Most parents send their children to preschool for social development and do not have a clear idea about what types of education they want for their children. B. Cognitive abilities of 3- to 5-year-old children are constantly changing. C. The children in the pre-schools that were studied had previously been exposed to another math enrichment program. D. Children are not really interested in enrichment programs in preschool. E. The cost factor needs to be specified and established before a large scale program can be undertaken. Ans : C 22. The symptoms of mental disorders are behavioral, cognitive, or emotional problems. Some patients with mental disorders can be effectively treated with psychotherapy, but it is now known that in some patients' mental disorders result from chemical imbalances affecting the brain. Thus, these patients can be effectively treated only with medication that will reduce or correct the imbalance. The argument depends on assuming which one of the following? A. Treatment by psychotherapy can produce no effective reduction in or correction of chemical imbalances that cause mental disorders. B. Treatment with medication always shows faster results for patients with mental disorders than does treatment with psychotherapy

C. Most mental disorders are not the result of chemical imbalances affecting the brain. D. Medication is always more effective in treating patients with mental disorders than is psychotherapy. E. Treatment with psychotherapy has no effect on mental disorders other than a reduction of the symptoms. Ans : A 23. Dear Editor: I feel obliged to comment on the unfair review you published last week written by Robert Duxbury. Your readers should know that Mr. Duxbury recently published his own book that covered the same topic as my book, which you asked him to review. It is regrettable that Mr. Duxbury should feel the need to belittle a competing work in the hope of elevating his own book. The author of the letter above makes her point by employing which method of argument? A. B. C. D. E.

Attacking the motives of the author of the unfavorable review. Attacking the book on the same topic written by the author of the review. Contrasting her own book with that written by the author of the review. Questioning the judgment of the author of the unfavorable review. Stating that her book should not have been reviewed by the author of a competing work.

Ans : A 24. The government of Zunimagua has refused to schedule free elections, release political prisoners, or restore freedom of speech; therefore, no more financial aid from the United States should be provided to Zunimagua. Which of the following is an assumption made in the argument above? A. Withdrawal of U.S. aid from Zunimagua will force a change in the policies of its government. B. The people of Zunimagua would be better off if their present despotic government were overthrown. C. The government of Zunimagua is dependent on continued U.S. aid for its existence. D. U.S. aid should be given only to countries willing to adopt policies in line with U.S. interests and goals. E. U.S. aid should be withdrawn from any country that refuses to operate its government along democratic lines. Ans : E 25. Many people argue that the death penalty deters murder. However, the notorious killer Ned Grandy deliberately moved to a state that imposes the death penalty just before embarking on a series of ferocious murders. Thus, it seems clear that the existence of the death penalty does not serve as a deterrent to murder.

The argument above may best be characterized as: A. B. C. D. E.

an appeal to emotion. a flawed analogy. a general conclusion based on a specific example. circular reasoning. an application of a general principle to a specific example.

Ans : C 26. Steve and JoAnne are both members of a certain club, though they are not speaking to each other and refuse to work with each other. Cecily, the club president, is appointing members to the fundraising committee, but she has resolved that she will not appoint anyone without his or her explicit consent. Steve tells Cecily, "I will not consent to appointment on that committee unless I know whether JoAnne is to be a member of it." And JoAnne says, "I will not consent to be a member of that committee unless I know whether Steve will be appointed to it." If all three of these people stick by these resolutions, then: A. Neither of them can be appointed to the committee. B. The situation described in the scenario cannot arise, because it is inherently incoherent. C. They must either both be appointed or both be left out. D. The committee may finally have one of them, both of them, or neither of them as members. E. Either one of them can be appointed, but not both. Ans : E 27. Russia's aggressive fishing in the prime fishing grounds of the Northern Pacific has led to a sharp decline in the populations of many fish and a general increase in the retail price of fish. This same pattern has occurred with far too many of our scarce vital natural resources, resulting in high prices for many products. It is likely then, that fish prices will continue to rise in the near future. In making the argument above, the author relies on all of the following assumptions EXCEPT: A. The scarcity of fish is a determining factor in its price. B. The decline in the number of fish available will result in higher prices for fish in stores. C. There will not be any substantial decrease in other costs involved in the fishing process that could keep the price of fish from increasing. D. Fish populations will not recover in the near future. E. Fishing practices can substantially influence the demand for fish. Ans : E

28. During the past year, Boz Corporation, a cigarette manufacturer, has engaged in a "corporate image" advertising campaign. One executive now urges that the advertising be extended for another year because profits have increased by 29% over the previous year. Another executive, however, is skeptical. She observes that the increases are typical for the industry over the past year, although none of their competitors have used corporate image advertising. The most accurate way of summarizing the second executive's point would be: A. She argues that the effect may not really be due to its supposed cause because there has not been a sufficient lapse of time between the cause and the effect. B. She argues that the assignment of a cause for this effect is premature, because there is as yet no well-established theory of such interactions. C. She argues that corporate image advertising is unprofitable, since it has evidently benefited competitors as much as the corporation that paid for it. D. She knows that effective advertising requires a constant influx of new ideas and approaches, and she argues that one year of corporate image advertising is enough for awhile. E. She argues that the effect may not be due to its alleged cause since the same effect is found elsewhere without that cause. Ans : E 29. The senate candidate expressed outrage that few judges have any background in technology, yet they try to resolve cases involving high tech companies. He stated that not one federal judge has a degree or any experience in computer technology. A promising response to this concern, arguing that things are not as bad as they might seem, could involve which of the following claims? A. Most of the public policy questions in this area are really about the morality and the value of scientific and technological developments. They do not require much technical understanding beyond that of a layperson. B. Computer scientists, by and large, have little interest in politics and public policy. It would be difficult to find scientists with the degree of commitment required for a serious contribution to the judicial system. C. There is a lack of people who are qualified in both technical and legal areas of expertise. D. There is very little opportunity for, and indeed little need for, technical expertise in the judicial branch. There is therefore almost no way for a technical specialist to rise through the ranks to a top-level position in government. E. The rewards of a life as a judge, in terms of both money and prestige, are not high enough to attract top-flight technical experts to this area. Ans : A 30. There has been a sharp increase in the subscription prices of many professional and scholarly journals in the past seven years. Many publishers ascribe the necessity for

these increases to the easy availability of photocopying facilities, which enable people simply to copy the articles they want rather than buying the journal. Which of the following, if it is true, would make this explanation more plausible? A. The great majority of professional and scholarly journals have a massive backlog of papers awaiting publication. B. Over the past five years there has been a substantial decline in the number of individual subscriptions to professional and scholarly journals, while library subscriptions have remained fairly stable. C. In the five years immediately preceding the price surge, there was a substantial decline in the number of individual subscriptions to professional and scholarly journals, while library subscriptions remained fairly stable. D. Many libraries have recently begun cutting back on subscriptions to professional and scholarly journals. E. In almost every field, several new professional and scholarly journals have begun publication in the past few years. Ans : C

Analytical Section : Logical Reasoning 31. Smoking in bed has long been the main cause of home fires. Despite a significant decline in cigarette smoking in the last two decades, there has been no comparable decline in the number of people killed in home fires. Each one of the following statements, if true over the last two decades, helps to resolve the apparent discrepancy above EXCEPT: A. Compared to other types of home fires, home fires caused by smoking in bed usually cause relatively little damage before they are extinguished. B. Home fires caused by smoking in bed often break out after the home's occupants have fallen asleep. C. Smokers who smoke in bed tend to be heavy smokers who are less likely to quit smoking than are smokers who do not smoke in bed. D. An increasing number of people have been killed in home fires that started in the kitchen. E. Population densities have increased, with the result that one home fire can cause more deaths than in previous decades. Ans : B 32. Mrs. Mason is gifted with psychic powers that enable her to foretell future events. In the past, Mrs. Mason has predicted such actual events as the election of President Clinton, the stock market crash of 1987, and the St. Louis Cardinals' 1982 World Series victory. These are just a few of Mrs. Mason's accurate predictions. The answer to which of the following questions would be most useful in evaluating the strength of the argument above?

A. What percentage of Mrs. Mason's predictions has come true? B. Could the election of President Reagan have been predicted without the help of psychic powers? C. What is the actual mechanism by which Mrs. Mason's psychic powers are supposed to operate? D. How long before the events in question did Mrs. Mason make her accurate predictions? E. Do most scientists accept the idea that the power to predict the future through psychic means really exists? Ans : A 33. An ingredient in coffee, known as RTC, has been found to inactivate common cold viruses in experiments. In previous experiments, researchers found that inactivated common cold viruses can convert healthy cells into cancer cells. It can be concluded that the use of coffee can cause cancer. Which one of the following, if true, most seriously weakens the argument? A. Several teams of scientists performed the various experiments, and all of the teams had similar results. B. The carcinogenic effect of RTC could be neutralized by the other ingredients found in coffee. C. When RTC kills common cold viruses it weakens the immune system, and it might thus diminish the body's ability to fight other viruses, including viruses linked to cancers. D. If chemists modify the structure of RTC, RTC can be safely incorporated into medications to prevent the common cold. E. To lessen the undesirable side effects of chemotherapy, the use of coffee has been recommended for cancer patients who are free of the common cold virus. Ans : B 34. Jack Bygrave is an executive at a major South African diamond company that produces 2% of the world's total annual diamond production. The CFO is anxious to maximize revenues and increase sales. Bygrave, however, believes that increased production would only drive down the world price of diamonds and lower revenues. Which of the following represents the logical flaw in Bygrave's reasoning? A. Jack connects the price of unrefined diamonds and the price of jewelryquality diamonds. B. He assumes that production goals are similar to financial goals. C. He assumes that the supply produced by a single company can significantly alter the aggregate supply for the market. D. He assumes that seasonal and long term supply are proportional. E. He correlates long-term and short-term demand. Ans : C

35. The crux of creativity resides in the ability to manufacture variations on a theme. If we look at the history of science, for instance, we see that every idea is built upon a thousand related ideas. Careful analysis leads us to understand that what we choose to call a new theme or a new discovery is itself always and without exception some sort of variation, on a deep level, of previous themes. If all of the statements in the passage are true, each of the following must also be true EXCEPT: A. A lack of ability to manufacture a variation on a previous theme connotes a lack of creativity B. No scientific idea is entirely independent of all other ideas. C. Careful analysis of a specific variation can reveal previous themes of which it is a variation. D. All great scientific discoverers have been able to manufacture a variation on a theme. E. Some new scientific discoveries do not represent, on a deep level, a variation on previous themes. Ans : E 36. Studies of fatal automobile accidents reveal that, in the majority of cases in which one occupant of an automobile is killed while another survives, it is the passenger, not the driver, who is killed. It is ironic that the innocent passenger should suffer for the driver's carelessness, while the driver often suffers only minor injuries or none at all. Which of the following is an assumption underlying the reasoning in the passage above? A. In most fatal automobile accidents, the driver of a car in which an occupant is killed is at fault. B. Drivers of automobiles are rarely killed in auto accidents. C. Most deaths in fatal automobile accidents are suffered by occupants of cars rather than by pedestrians. D. Auto safety experts should increase their efforts to provide protection for those in the passenger seats of automobiles. E. Automobile passengers sometimes play a contributing role in causing auto accidents. Ans : A 37. The editors of Business Today magazine conducted a poll of its readers regarding the proposed increase in the rate of income tax paid on profits from the sale of stocks. More than 60% of the readers opposed the proposed tax. The editors announced that the majority of Americans opposed any increase in the tax on profits from stock sales. Which one of the following statements, if true, would most weaken the editor's conclusion?

A. Some readers of Business Today magazine are citizens of countries other than the United States. B. Decisions concerning the income tax laws are made by the Congress rather than directly by the people. C. Most of those who earn profits from stock sales are wealthy and can afford to pay higher taxes. D. The viewpoints of the vast majority of the readers of Business Today magazine differ from the views of most Americans. E. Not all readers of Business Today magazine responded to the editors' poll. Ans : D 38. Several movie versions of Charles Dickens' Tale of Two Cities have been made. The original movie version made in 1939 is the best because it is closest in spirit to the original novel. An underlying assumption of the argument above is that a movie based on a novel should: A. reflect the director's original interpretation of the main themes of the novel. B. accurately depict the time and place in which the novel is set. C. feature actors and actresses who closely resemble the characters in the novel both in body and spirit. D. faithfully render the details of the plot from the narrator's point of view. E. capture the true meaning and intention of the novel. Ans : E Following are some CAT sample general awareness questions. 1. Which company's famously advertised vision statement is 'The Network is the Computer'? A. Cisco Systems B. Lucent Technologies C. Sun Microsystems D. Nortel Networks Answer : C 2. The software company I-flex Solutions was originally a division of which famous financial services company? A. Citicorp B. ICICI C. HSBC D. ABN Amro Bank Answer : A

3. Which former advertising personality has recently been named the Undersecretary of State for 'public diplomacy and public affairs' of the US in an exercise to rebrand the US following the September 11 terrorist strikes? A. Charlotte Beers B. Jay Chiat C. Martin Sorrell D. George Lois Answer : A 4. Which premium international range of luggage was recently launched in India by BlowPlast? A. Samsonite B. Strolley C. American Tourister D. Delsey Answer : D 5. This year saw the launch of Yahoo!, Amazon.com and the famous launch of the Orange mobile phone service in the UK. Which year was this? A. 1992 B. 1994 C. 1995 D. 1993 Answer : B 6. Henry Ford revolutionised the car market with the first mass- produced car, the Ford Model T. In which year was it launched? A. 1924 B. 1912 C. 1908 D. 1897 Answer : C 7. Which company owns the beer brands Haywards 2000, Hi-Five and Lal Toofan? A. United Breweries B. Millennium Alcobev C. Shaw Wallace D. Mohan Meakins Answer : C 8. Which company owns the footwear brand - Stryde? A. Woodland B. Bata C. Tata International D. Hindustan Lever

Answer : C 9. It was the brand that made David Ogilvy famous. Which shirt brand's ads had the famous man with an eye patch in the 1950s, which catapulted David Ogilvy to fame? A. Van Heusen B. Arrow Shirts C. Hathaway D. Dockers Answer : C 10. Which TV channel has been in the news for its coverage of the Bin Laden crisis, especially for having carried all broadcasts of Osama Bin Laden to audiences in West Asia? A. Star Asia B. BBC Asia C. Al Jazeera D. Khaleed Times Answer : C

General Awareness 11. In the case of a test tube baby A. Fertilization takes place inside the test tube B. Development of the baby takes place inside the test tube C. Fertilization takes place outside the mother's body D. Unfertilized egg develops inside the test tube Answer : C 12. Who elects the President of India? A. Lok Sabha B. Rajya Sabha C. People of India D. Parliament and State Assemblies Answer : D 13. The Fundamental Duties were incorporated in the Constitution of India by the A. First Amendment B. Tenth Amendment C. Thirty-second Amendment D. Forty-second Amendment Answer : D

14. The speed of light will be minimum while passing through A. glass B. vacuum C. air D. water Answer : A 15. The time taken by the Sun to revolve around the center of our galaxy is A. 50 mn years B. 100 mn years C. 250 mn years D. 365 mn years Answer : C 16. The oldest monarchy in the world is that of A. Nepal B. U K C. Spain D. Japan Answer : D 17. The Vice-President is elected by an electoral college consisting of A. only elected members of both Housed of Parliament B. only Rajya Sabha members C. all members of Parliament and State Legislative Assemblies D. all members of Lok Sabha and Rajya Sabha Answer : D 18. In which one of the following states of India is it legal for a Hindu male and illegal for a Muslim male to have more than one living wife? A. Nagaland B. Goa C. Himachal Pradesh D. Arunachal Pradesh Answer : B 19. Where did Buddha die? A. Lumbini B. Kusinagara C. Pavapuri D. Magadha Answer : B

20. What was the main difference between the Indus Valley Civilization and Vedic Civilization? A. Indus Valley Civilization was urban, while the Vedic Civilization was rural. B. "Pipal" tree was worshiped in Indus Valley Civilization, while "Burgad" tree was worshiped in Vedic Civilization. C. The main emphasis in Indus Valley Civilization was on trade while in the Vedic Age was on religion. D. Indus Valley Civilization believed in non-violence while Vedic Civilization had no hard and fast rules about violence. Answer : A

General Awareness 21. All of the following are constituents of RNA molecule except A. Adenine B. Guanine C. Thymine D. Uracil Answer : C 22. The father of modern biology is A. Aristotle B. Darwin C. Mendel D. Vesalius Answer : A 23. Which of the following seeds will normalize blood sugar level? A. Coriander B. Mustard C. Cumin D. Fenugreek Answer : D 24. The point at which solid, liquid and gaseous forms of a substance co-exist is called A. sublimation point B. distillation point C. triple point D. melting point Answer : C 25. Which among the following substances is used as a lubricant? A. Quartz B. Silica

C. Graphite D. Nickel Answer : C 26. The weight of an object will be minimum when it is placed at A. The North Pole B. The South Pole C. The Equator D. The center of the Earth Answer : D 27. Earth quake waves travel fastest in A. Soil B. Molten rock C. Water D. Flexible rock Answer : C 28. The angle between the geographical meridian and magnetic meridian is called A. Angle of dip B. Angle of declination C. Angle of inclination D. None of the above Answer : B 29. What is the currency of Mexico? A. Lira B. Peso C. Mexican dollars D. Krones Answer : B 30. The Essar group of companies has been promoted by A. Ruias B. Ambanis C. Goenkas D. Kanorias Answer : A

General Awareness 31. One of the following liquor brands is not owned by United Breweries A. Kalyani Black Label

B. Blue Riband C. McDowells D. Bagpiper Answer : D 32. Amtrex air conditioners has a technical collaboration with A. Sanyo B. Mitsubishi C. Hitachi D. Carrier Answer : C 33. Marlboro Cigarette is owned by A. ITC B. Godfrey-Philips C. Philip Morris D. British American Tobacco Answer : C 34. Who is the CEO of Microsoft? A. Bill Gates B. Paul Allen C. Larry Ellison D. Steve Ballnes Answer : D 35. One of the following is not an Insurance Company A. ICICI Prudential B. HDFC Natwest C. OM Kotak Mahindra D. Birla Sun Life Answer : B 36. What is the currency of Portugal? A. Escudo B. Guilder C. Mark D. Schilling Answer : A 37. One of the following personalities is not associated with the Indian auto sector A. Suresh Krishna B. Venu Srinivasan C. Anand Mahindra

D. Venugopal Dhoot Answer : D 38. One of the following companies is not in the area of air-conditioning & refrigeration A. BPL B. Thermax C. Lloyd D. Carrier Aircon Answer : C 39. How many countries are a part of the European union? A. 18 B. 15 C. 21 D. 12 Answer : B 40. Who is the attorney general of United States of America? A. Ashcroft B. Powell C. Rumsfeld D. Cheney Answer : A

General Awareness 41. When was the generic domain name (Top Level Domain - TLD) .com introduced? A. 1989 B. 1985 C. 1994 D. 1991 Answer : B 42. Who is the prime minister of Israel? A. Benajamin Netanyahu B. Ariel Sharon C. Shaul Mofaz D. Goldamyer Answer : B

43. What was the significant about the purchase of a kilo of lychees on the French island of Reunion, located in the Indian Ocean? A. They were the first lot of lychees exported from India. B. It was the first official purchase using the new currency Euro. C. It was done by to commemorate the release of the new France with Princess Diana's image on it. D. None of these Answer : B 44. Which brand had the highest number of Web searches in 2001? A. Google B. Marlboro C. Intel D. Play Station Answer : D 45. What is common to all of the following names - Eduardo Camano, Adolfo Rodriguez Saa, Ramon Puerta and Fernando de la Rua? A. They were all former Presidents of Argentina. B. They are the dreaded drug lords of South America C. They own together 70% of the world's Silver mines. D. None of these Answer : A 46. The video games X box is a product of A. Sega B. Sony C. Intel D. Microsoft Answer : D 47. It was acknowledged as the second-most dangerous computer virus in history, after the Love Bug virus. Name this virus, which struck in 2001 A. Melissa B. Code Red C. C-Brain D. Major Domo Answer : B 48. What does the letters XP stand for in the product Microsoft XP? A. Extended product B. Extra Pampering C. Experience D. Entry level product

Answer : C 49. Which business and media tycoon won the elections to become Head of State in Italy amidst widespread clouds of scandal? A. Guillani Giovanni B. Joe Pacci C. Antonio Machiaveli D. Silvio Berusconi Answer : D 50. Which city is hosting the 14th Asian Games in 2004? A. Manila B. Busan C. Beijing D. Bangkok Answer : B

Quantitative Section : Data Interpretation Questions 1 - 5 refers to the following table: PROFILE OF CONGRESS IN YEAR X (total membership: 535) House of Representatives 292 143 435 418 17 27 77 48 255 107 18 4 51 House of

Senate Party Democratic Republican TOTAL Sex Male Female Age Youngest Oldest Average (arithmetic mean) Religion Protestant Catholic Jewish Mormon Other

62 38 100 100 0 34 80 54 69 12 5 3 11 Senate

Representatives 215 81 45 14 22 24 2 1 0 6 25 17 2 4

Profession Lawyer Business Executive

or Banker Educator Farmer or Rancher Career Government Official Journalist or Communications Executive Physician Veterinarian Geologist Worker or Skilled Tradesperson Other Ethnic Group Black American Asian American Hispanic American

63 15 6 6 0 4 0 1 2 0 3 1 3 0

1. In the Senate, if 25 male members were replaced by 25 female members, the ratio of male members to female members would be A. 4 to 1 B. 3 to 1 C. 3 to 2 D. 2 to 1 E. 1 to 1 Ans : B 2. Approximately what percent of the members of Congress are lawyers? A. 63% B. 58% C. 56% D. 52% E. 49% Ans : D 3. If 5 senators are Catholic Democrats, how many senators are neither Catholic nor Democratic? A. 79 B. 74 C. 69 D. 31 E. 21

Ans : D 4. If all lawyers and all women in the House of Representatives vote for the passage of a bill, how many more votes will be needed for a majority? A. 435 B. 220 C. 3 D. 0 E. It cannot be determined from the information given. Ans : E 5. Which of the following can be inferred from the information given in the chart? I.More than 80 percent of the men in Congress are members of the House of Representatives. II.The percent of members who are categorized as farmers or ranchers is greater for the House of Representatives than for the Senate. III.The median age in the Senate is 57. A.I only B.II only C.III only D.I and II E.I and III Ans : A

Quantitative Section : Data Interpretation Questions 6 - 7 refers to the following table: PERCENT CHANGE IN DOLLAR AMOUNT OF SALES IN CERTAIN RETAIL STORES FROM 1977 TO 1979 Percent Change Store P Q R S T

From 1977 to 1978 +10 -20 +5 -7 +17

From 1978 to 1979 -10 +9 +12 -15 -8

6. In 1979, for which of the stores was the dollar amount of sales greater than that of any of the others shown?

A. B. C. D. E.

P Q R S It cannot be determined from the information given.

Ans : E 7. In store T, the dollar amount of sales for 1978 was approximately what percent of the dollar amount of sales for 1979? A. 86% B. 92% C. 109% D. 117% E. 122% Ans : C Questions 8 - 9 refers to the following Figure:

8. Of every dollar received by the federal government, how much (in cents) is from coporate sources? A. 32 B. 70 C. 30 D. 35 E. 29 Answer : 1 9. what percentage of the federal revenue is derived from borrowings? A. 0.2% B. 0.02% C. 2.7% D. 1.2% E. 2.5%

Answer : 3

Quantitative Section : Data Interpretation Questions 10 - 11 refers to the following table: DIRECTIONS: The following question are based on the bellow table, which shows per capita Mean Expenditure, Per capita Food expenditure, Number of Households and Per capita cereal consumption, in both quantity and value, for different expenditure classes of rural India. The sampled 41597 households are divided into 12 expenditure classes, starting from less than Rs.65 per month per capita and ending at more than Rs.385 per capita per month.

10. According to the results of this sample survey, what is the proportion of total expenditure on food to total expenditure for all the sampled households taken together? A. 58% B. 36.7% C. 63.3% D. 71% E. Cannot be determined Answer : 3 11. What is the difference, approximately, between the gross expenditure of the sampled households in the Rs.95-110 expenditure class and in the Rs.180-215 expenditure class? A. 372000 B. 448000 C. 496000 D. 93.8 E. 52.3 Answer : A Questions 12 - 13 refers to the following Graph:

GRAPH SHOWS EXPENDITURE ON ARMS BY DIFFERENT COUNTRIES (VALUE IN DOLLARS '000 MILLIONS)

12. The amount spent by country C in 1983 is what percentage more than the amount spent by Countries A and B together in 1977? (Find approximately) A. 50% B. 179% C. 75% D. 13% E. 70% Answer : C 13. Which of the following statements must be true? i.Country A spends minimum amount of its budget on arms. ii.Throughout, Country C has spent the maximum amount on arms during the years shown. iii.An examination of the information for the last 3 years reveals that generally all 3 countries are reducing their expenditure on arms. A.i only. B.i and ii only C.i and iii only D.ii and iii only E.None of the statements above. Answer : E

Related Documents

Quantity A
May 2020 0
Physical Quantity
May 2020 19
Comment Quantity
August 2019 20
Quantity Armature
June 2020 4